You are on page 1of 97

TAXATION QUIZZER PART

BASIC PRINCIPLES OF TAXATION

1. Which theory in taxation states that without taxes, a


government would be paralyzed for lack of power to activate
and operate it, resulting in its destruction?
a. Power to destroy
theory b. Lifeblood
theory
c. Sumptuary theory
d. Symbiotic doctrine

2. The actual effort exerted by the government to effect the


exaction of what is due from the taxpayer is known as
a.

Assessment. b.
Levy.
c. Payment.
d. Collection.

3. Although the power of taxation is basically


legislative in character, it is NOT the function of Congress
to
a. Fix with certainty the amount of
taxes. b. Collect the tax levied under
the law.
c. Identify who should collect the tax.
d. Determine who should be subject to the tax.

4. An example of a tax where the concept of progressivity


finds application is the
a. Income tax on individuals.
b. Excise tax on petroleum products.
c. Value-added tax on certain
articles. d. Amusement tax on boxing
exhibitions.

5. Ligaya Educational Foundation, Inc., a stock


educational institution organized for profit, decided to lease
for commercial use a 1,500 sq. m. portion of its school.
The school actually, directly, and exclusively use the rents
for the maintenance of its school buildings, including payment of
janitorial services. Is the leased portion subject to real
property tax?
a. Yes, since Lualhati is a stock and for profit
educational institution.
b. No, since the school actually, directly, and
exclusively
used the rents for educational purposes.
c. No, but it may be subject to income taxation on the
rents it receives.
d. Yes, since the leased portion is not actually, directly,
and
exclusively used for educational purposes.

6. Which among the following concepts of taxation is the


basis for the situs of income taxation?
a. Lifeblood doctrine of
taxation b. Symbiotic relation in
taxation
c. Compensatory purpose of
taxation d. Sumptuary purpose of
taxation

DONOR’S TAX
7. Which of the following transactions is deemed a taxable gift?
a. Condonation or remission of a debt
b. Sale of residential house and lot for less than adequate
and full consideration in money or money’s worth
c. Both (a) and (b)
d. Neither (a) nor (b)

8. Which of the following statements relative to donor’s tax


is false?
a. The spouses shall file separate donor’s tax returns
where the thing donated is common property.
b. Each parent shall be entitled to the P10,000 exemption
on account of marriage of a child.
c. Exemptions and deductions cannot be claimed where the
30%
tax rate on stranger is applicable.
d. None of the foregoing.

9. The spouses Esme and Carlisle wanted to donate a parcel of land


to their son Edward who is getting married in December,
2016. The parcel of land has a zonal valuation of P420,000.00.
What is the most efficient mode of donating the property?
a. The spouses should first donate in 2016 a portion of
the property valued at P20,000, then spread the P400,000
equally for 2017, 2018, 2019 and 2020.
b. Spread the donation over a period of 5 years by the
spouses donating P100,000 each year from 2016 to 2020.
c. The spouses should each donate a P110,000 portion of
the
value of the property in 2016 then each should donate
P100,000 in 2017.
d. The spouses should each donate a P100,000 portion of
the value of the property in 2016, and another P100,000
each in
2017. Then, in 2018, Esme should donate the remaining
P20,000.

10. Exempted from donor’s taxation are gifts


made a. For the use of the barangay.
b. In consideration of marriage.
c. To a school which is a stock corporation.
d. To a for-profit government corporation.

11. Caroline donated P110,000.00 to her friend Vicky who was


getting married. Caroline gave no other gift during the
calendar year. What is the donor's tax implication on Caroline’s
donation?
a. The P100,000 portion of the donation is exempt since
given in consideration of marriage.
b. A P10,000 portion of the donation is exempt being a
donation in consideration of marriage.
c. Caroline shall pay a 30% donor's tax on the P110,000
donation.
d. The P100,000.00 portion of the donation is exempt under
the rate schedule for donor's tax.

12. A non-stock, non-profit school always had cash flow


problems, resulting in failure to recruit well-trained
administrative personnel to effectively manage the school. In
2017, Don Leon donated P100 million pesos to the school,
provided the money shall be used solely for paying the salaries,
wages, and benefits of administrative personnel. The donation
represents less than 10% of Don Leon's taxable income for the
year. Is he subject to donor's taxes?
a. No, since the donation is actually, directly, and
exclusively used for educational purposes.
b. Yes, because the donation is to be wholly used for
administration purposes.
c. Yes, since he did not obtain the requisite NGO
certification before he made the donation.
d. No, because the donation does not exceed 10% of his
taxable income for 2017.

13. What law shall govern the imposition of donor’s tax?


a. The law in force at the time of perfection of the
donation b. The law in force at the time of completion of
the donation
c. The law in force at the time of perfection or
completion depending upon the agreement of the parties
d. None of the choices

14. Andy, married, donated a land commonly owned by him and her
spouse worth P500,000 to her friend Joan. Only Andy signed
the deed of donation. Joan assumed P200,000 unpaid mortgage
on the property. How much is the donor’s tax due?
a. P6,000 c.
b. P90,000 d. P1,000
P45,000

500k - 200k = 300k x 30% = 90,000

15. Dondie, resident citizen, made the following donations on April


28 of the current year:
 To his sister, Donna,P175,000 worth of property situated in
Paris, France. The donor’s tax paid is P40,000.
 To Dara, his girlfriend in the Philippines, jewelry valued
at
P225,000.
 To International Rice Research Institute, cash amounting to
P50,000.
The donor’s tax due after tax credit is –
a. P69,000 c. P29,000
b. P38,813 d. P5,625
225,000 x 30% = 67,500 + (175,000 - 100,000 x 2% Tabular) = 69,000
- (69,000 x 175k/400k) = 38,813

ESTATE TAX
16. Which of the following is not a part of the gross estate?
a. Conjugal property
b. Community property
c. Exclusive property of the decedent
d. Exclusive property of the surviving spouse

17. Who among the following transferors is not liable for estate
tax on the property transferred during his lifetime?
a. The testator who bequeaths property to his heirs in a
last will and testament executed and probated during his
lifetime
b. The donor who reserves his right to amend or revoke the
donation of property in favor of the donee
c. The donee of an appointed property who is required under
a
power of appointment to transfer such property upon death
to his eldest child
d. The transferor of personal property who sold it for
insufficient consideration

18. Which of the following properties of the spouses will be part


of common properties under a regime of Conjugal Partnership of
Gains?
a. Land inherited during the
marriage b. Fruits of land inherited
c. Jewelry inherited during the
marriage d. Building donated before
marriage

19. Which of the following items is not considered as a


“special deduction” in computing the taxable net estate of the
decedent?
a. Vanishing deduction
b. Medical expenses
c. Standard deduction
d. Family home allowance

20. When the payment of estate tax will cause undue hardship upon
the heirs or the estate which is undergoing judicial settlement
before the court, the BIR Commissioner may grant an
extension for a period not exceeding:
a. 5 years
b. 3 years
c. 2 years
d. 1 year

Next three (3) questions are based on the following:


Orland, married resident citizen, died on August 20, 2016. The
estate reported the following assets and deductions:
Conjugal Properties:
Fishpond, Bulacan P1,500,000
Family Home, Makati 1,500,000
Cash in bank 900,000
Exclusive Properties of Orland: P400,000
Land, inherited from his father
who died on July 20, 2012. The
value of land at the time of inheritance
was P210,000. The land was mortgaged for
P30,000 which was unpaid at the time
of death of his father,P10,000 of
which was paid by Orland before he
died.

Land, donated on February 14, 2013 600,000


by his mother who died on
November 2,
2014. The value of the land when
donated was P500,000 while upon
death of his mother was P400,000.

Exclusive Properties of Wife: 2,000,000


Farm in Laguna, acquired
before marriage

Deductions claimed:
Funeral expenses 250,000
Fire loss of apartment (occurred 80,000
4 months after death)
Bad debts (represents unpaid 100,000
receivable from Bert, an insolvent)
Mortgage on inherited land 30,000
Vanishing deduction on 40,000
inherited land
Vanishing deduction on donated land 20,000
Standard deduction 2,000,000

21. The gross estate is:


P4,000,000 c. P7,000,000
a. P4,710,000 d. P5,000,000
b.
22.

23. The net taxable estate is:


a. P819,200 c. P829,200
b. P804,200 d. P579,200
SOLUTION:
Properties Exclusive Conjugal Total
Fishpond, Bulacan P1,500,000 1,500,000
Family Home, Makati 1,500,000 1,500,000
Cash in bank 900,000 900,000
Land, inherited from Father
died on July 20, 2012. 400,000 400,000
Land, Donated from Mother
on Feb 14, 2013. 600,000 600,000
Claims to Insolvent 100,000 100,000
GROSS ESTATE 1,000,000 4,000,000 5,000,000
Allowable Deductions:
Funeral Expenses (200,000) (200,000)
Fire Loss (80,000) (80,000)
Bad Debts (100,000) (100,000)
Mortgage on Land (20,000) (20,000)
Vanishing Deductions* (220,800) (220,800)
Net Estate before Special
Deductions 759,200 3,620,000 4,379,200
Special Deductions:
Family Home (1,500,000 x 1/2) (750,000)
Special Deduction (1,000,000)
Share of Surviving Spouse (1,810,000)
NET TAXABLE ESTATE 819,200

*VANISHING DEDUCTIONS:
Land Inherited by Father:
Value at the time of Death 210,000
Less: Mortgage Paid (10,000)
Initial Basis 200,000
Pro rata: 200/5000 x 400,000 (16,000)
Final Basis 184,000
Vanishing Rate (4 years but not more 5 yrs) 20%
Vanishing Deduction 36,800

Land Donated by Mother:


Value at the time of Donation 500,000
Initial Basis 500,000
Pro rata: 500/5000 x 400,000 (40,000)
Final Basis 460,000
Vanishing Rate (3 years but not more 4 yrs) x 40%
Vanishing Deduction 184,000
TOTAL VANISHING DEDUCTIONS 220,800

**If TRAIN Law is applied the Estate tax is P 49,152 (P819,200 x 6%)
24. Abe, married resident alien, died on January 15, 2017. She
left the following properties, expenses and obligations:

Community properties, Philippines P5,000,000


(including family home valued at
P1,800,000)
Community properties, Abroad 2,000,000
Exclusive properties, Philippines 3,000,000
Actual funeral expenses 300,000
Judicial expenses 200,000
Medical expenses (incurred w/in 1yr. before death) 600,000
Devise to National Gov’t 50,000
Legacy to Local Gov’t 70,000

The net taxable estate is:


a. P3,780,000 c. P3,580,000
b. P3,680,000 d. P3,530,000

RESIDENT ALIEN
Particulars Exclusive Community
Total
All Properties w/i & w/o 3,000,000 7,000,000
10,000,000
Funeral Expense (200,000)
(200,000) Judicial Expense
(200,000) (200,000) Transfers (50,000 + 70,000) (120,000)
(120,000) Gross Estate 2,880,000 6,600,000
9,480,000
Share of Surviving Spouse
(3,300,000) Medical Expenses
(500,000) Family Home (1/2 of 1,800,000)
(900,000) Standard Deductions
(1,000,000) Net Estate
3,780,000

25. Based on the above problem, if the decedent is a non-resident


alien, how much is the net taxable estate?
a. P2,755,000 c. P5,220,000
b. P2,880,000 d. P5,380,000

NON - ALIEN
RESIDENT Exclusive Community Total
All Properties w/i Only 3,000,000 5,000,000 8,000,000
Funeral Expense 200,000
Judicial Expense 200,000
Allowable Deduction 400,000 x 8M/10M (320,000) (320,000)
Transfers(50,000 + 70,000) (120,000) (120,000)
Gross Estate 2,880,000 4,680,000 7,560,000
Share of Surviving Spouse (2,340,000)
Net Estate 5,220,000
26. Arthur, Filipino, married died leaving the following estate:

Car acquired before marriage by Arthur P 300,000


Car acquired before marriage by wife 450,000
House and lot acquired during marriage 1,500,000
Jewelries of wife 100,000
Personal properties inherited by Arthur during 250,000
marriage
Benefits from SSS 50,000
Retirement benefits 150,000
Proceeds of group insurance taken by his employer 75,000
Land inherited by the wife during marriage 1,000,000
Income earned from the land inherited by wife 200,000
(25%
of which was earned after death)

16

Car acquired before marriage by Arthur P 300,000


House and lot acquired during marriage 1,500,000
Personal properties inherited by Arthur during marriage 250,000
Income earned from the land inherited by wife
(25%
Gross of which was earned after death) 150,000
Estate 2,200,000

27. Based on the preceding number, the gross estate if the


property relationship is absolute community of property is:
a. P2,600,000 c. P1,950,000
b. P3,600,000 d. P2,500,000

Car acquired before marriage by Arthur P 300,000


Car acquired before marriage by wife 450,000
House and lot acquired during marriage 1,500,000
Jewelries of wife 100,000
Personal properties inherited by Arthur during marriage
Gross Estate 250,000
2,600,000

VAT

28. LBJ made the following sales during the 12-month


period:

Sales, VAT taxable transactions P1,500,000


Sales, VAT zero-rated transactions 400,000
Sales, VAT exempt transactions
Total 100,000
P2,000,000

Which of the following statements is correct?


a. LBJ may not register under the VAT system because his sales
from
VAT taxable transactions did not exceed P1,919,500.
b. LBJ may not register under the VAT system because his sales
from
VAT taxable and zero-rated transactions did not exceed
P1,919,500.
c. LBJ is required to register because his total 12-month sales
exceeded P1,919,500.
d. None of the foregoing.

29.
a Philippines
b. Common carriers transporting passengers by sea within
the
Philippines
c. Common carriers transporting passengers by land within
the
Philippines
d. Common carriers transporting cargoes by air within the
Philippines

30. Which statement is correct about value-added tax on goods


or properties sold?
a. It is based on gross sales and not on net sales;
b. May be due even if the goods or properties were not
actually
sold;
c. Does not cover goods exported;
d. It forms part of the selling expense of the trader.

31. For value-added tax purposes, which of the following


transactions of a VAT-registered taxpayer may not be zero-rated?
a. Export sales
b. Foreign currency denominated sales
c. Sale of goods to the Asian Development Bank
d. Sale of goods to an export oriented enterprise

32. A subdivision developer sold five (5) residential house and


lots, each to different vendees, for P3,000,000 per lot, or
a total sales of P15,000,000 for the taxable period.
These sales shall be classified
as:
a. 12% VAT transactions
b. 0% VAT transactions
c. VAT exempt transactions
d. None of the foregoing

**3,199,000 each is the


threshold

33. CP operated a retail business that had been generating sales


not exceeding the threshold for VAT exempt persons.
However, he desires to be registered under the VAT system for
the first time in order to benefit from input tax credits.
What benefit may CP be entitled to once he registers under the
VAT
system?
a. Tax refund
b. Presumptive input tax credit
c. Transitional input tax
credit d. None of the foregoing
34. What institution is required to deduct and withhold a final VAT
of
5% on the purchase of goods or services subject to VAT?
a. National government or any political subdivision
thereof b. Government-owned or controlled corporations
c. Both (a) and (b)
d. Neither (a) nor (b)

35. In the value-added tax on sale of services, the output tax


is computed:
a. On the billings of the month
b. On collections of the month on all billings made
c. On the contract price of contracts completed during the
taxable period
d. Only and strictly on labor performed under the contract
for services

36. Which statement is wrong?


a. There is a transitional input tax from purchases of goods
or properties;
b. There is a transitional input tax from purchases of
services;
c. There is a transitional input tax from purchases of
materials;
d. There is a transitional input tax from purchases of
supplies.

37. Which of the following statements is correct on the


inventory balance in the financial statements at any given
date of a VAT- registered person?
a. Balance, net of input taxes
b. Balance, inclusive of input taxes
c. Balance on which the transitional input tax is computed
annually
d. Balance where the VAT thereon may be calculated by
multiplying it by 12%

38. Genson Distribution Inc., a VAT taxpayer, had the following


data in a month:

Cash sales P200,000


Open account sales 500,000
Consignment:
0 to 30 days old (on which there
were remittances from consignees
of
P200,000) 600,000
31 to 60 days old 700,000
61 days old and above 900,000
How much is the output tax?
a. P348,000 c. P264,000
b. P216,000 d. P108,000
Cash sales P200,000
Open account sales 500,000
Consignment:
0 to 30 days old (on which there were
remittances from consignees of P200,000) 200,000
61 days old and above 900,000
Total VATABLE SALES 1,800,000
VAT RATE 12%
OUTPUT VAT 216,000

42. The financial records of Benz Corp., a VAT-registered


taxpayer, for the taxable year 2016 disclosed the following:
Local 1,500,000
Export sales
Sales to private entities 500,000
Local sales to government 800,000

How much is the total sales subject to value-added tax?


a. P2,800,000 c. P2,000,000
b. P2,300,000 d. P1,500,000

Local sales to private entities 1,500,000


Export Sales 500,000
Local sales to government 800,000
Total VATABLE SALES 2,800,000

43. Mantika Corp., a VAT-registered Corp., is a producer of


cooking oil from coconut and corn. It had the following data for
the month of January 2017:
Sales, gross of VAT P 784,000
Corn & Coconut, 12-31-16 50,000
Purchases of Corn & Coconut 330,000
Corn & Coconut, 1-31-17 20,000
Purchases from VAT suppliers, VAT included:
Packaging Materials 56,000
Supplies 16,800
The value-added tax payable for the month:
a. P56,060 c. P60,650
b. P54,900 d. P63,000
Sales, gross of VAT P 784,000
Output TAX 84,000

Purchases of Corn & Coconut (330,000 x4%) (13,200)


Purchases from VAT suppliers, VAT included:
Packaging Materials 56,000
Supplies 16,800 72,800 x3/28 (7,800)
VAT PAYABLE 63,000
44. Bunga Inc., a VAT taxpayer, is engage in the business
of processing of fruits. Its data on sales and purchases
for the month of August are provided below:
Sales P200,000
Purchases:
Fresh Fruits 30,000
Raw sugarcane 12,000
Tin Can, gross of VAT 12,320
Paper Labels, net of VAT 5,000
Cardboard for boxes, net of VAT 8,000
Freight, gross of VAT (50% still unpaid) 10,080

How much is the value-added tax payable?


a. P20,580 c. P19,380
b. P18,900 d. P20,100

Sales P200,000 Output Tax 24,000


Purchases:
Fresh Fruits 30,000
Raw sugarcane 12,000
Tin Can, gross of VAT 12,320 Input Tax (1,320)
Paper Labels, 5,000
Cardboard for net of
boxes, of VAT 8,000 Input
Input Tax
Tax (
( 600)
960)
net
Freight, gross of VAT
(50% still unpaid) 10,080 Input Tax ( 540)
VAT PAYABLE 20,580

45. Bahay Kubo Inc. is a real estate dealer. Details of its


sales during the year showed the following:
Date of sale June 2, 2017
Consideration in the deed of sale P 5,000,000
Fair market value in the assessment rolls 4,800,000
Zonal Value 5,200,000
Schedule of payments:
June 2, 2017 1,000,000
June 2, 2018 2,000,000
June 2, 2019 2,000,000

How much is the output tax to be recognized for the June 2,


2018 payment?
a. P0 c. P249,600
b. P124,800 d. P624,000

**Zonal Value 5,200,000 x 12% = 624,000 x 2M/5M =


249,600 Output Tax for 2018

46. Assuming that the scheduled payment on June 2, 2017 is


P2,000,000, how much is the output tax to be recognized for the
June 2, 2019 payment?
a. P0 c. P249,600
b. P124,800 d. P624,000
Zero as in 0 for the sale will no longer qualify as
Installment
Sales.

47. Mr. Karpentero, a vat-registered building contractor, has the


following data on gross receipts in a month, any tax not
included:

 From Mr. A, a private property owner, final payment on


the contract price, net of 5% agreed retention fee of
P2,850,000

 From Mr. B, a payment of 5% retention on the contract


price previously made by him P100,000

 From Mr. C, for materials used in the construction


500,000

How much is the output tax?


a. P414,000 c. P72,000
b. P342,000 d. P62,000

Final Payment on Contracts 2,850,000


Retention 100,000
Materials 500,000
TOTAL Receipts 3,450,000
VAT Rate 12%
Output VAT 414,000

48. COC Inc., in its first month of operation, and as a VAT


taxpayer, purchased various fixed assets. Purchases of fixed
assets in the first month were as follows:

Light equipment, with a useful life of 3 years P 300,000


Heavy equipment, with a useful life of 10 years 4,000,000
How much is the input tax available for the month?
a. P516,000 c. P480,000
b. P9,000 d. P8,600

Light Equipment 300,000/36 12% 1,000


Heavy Equipment 4,000,000/60x x 12% 8,000
Total Input VAT 9,000

49. Kusina Co., had its kitchen assembled by a VAT taxpayer. It


took six months for the contractor to finish the work. Kusina Co.
purchased materials in July from VAT suppliers at a cost
of P500,000, VAT not included. Payment to the contractor in July
2017 on the Construction in Progress, VAT not included was:

On contractor’s billing in June P100,000


On contractor’s billing in July 70,000
The input tax available in July is:
a. P0 c. P60,000
b. P80,400 d. P20,400

Materials from VAT Supplier 500,000


On contractor’s billing in June 100,000
On contractor’s billing in July 70,000
Total 670,000
VAT Rate 12%
Output Tax 80,400

50. Data from the books of accounts of a VAT taxpayer for


February: Domestic Exports
Sales P 2,000,000 8,000,000
Purchases:
From VAT Suppliers:
Goods for sale 600,000 2,400,000
Supplies & services 90,000 360,000
From Suppliers Paying percentage tax:
Goods for sale 100,000 1,500,000
Supplies & services 20,000 80,000
If the input taxes attributable to zero-rated sales are claimed
as tax credit, the net value-added tax refundable is:
a. P136,000 c. P145,000
b. P203,924.70 d. P174,000

Output Tax (2,000,000 x 12%)


240,000
Input Tax on Domestic Sales (690,000 x 12%)
(82,800) Input Tax on Zero Rated (2,760,000 x 12%)
(331,200) Refundable VAT
(174,000)

INCOME TAXATION
51. C. Lee, Chinese national, arrived in the Philippines on January
1,
2012 to visit his Filipina paramour. He planned to stay in
the country until December 31, 2016, by which time he would go
back to his legal wife and family in China. C. Lee derived
income during his stay here in the Philippines.
For the taxable year 2012, C. Lee shall be classified as
a:
a. Resident alien
b. Non-resident alien engaged in trade or business in the
Philippines
c. Non-resident alien not engaged in trade or business in
the
Philippines
d. Special alien employee

52. The following individual taxpayers are subject to the


graduated income tax rates of 5%-32%, except
a. Filipino citizens
b. Resident aliens
c. Non-resident alien engaged in trade or business in the
Philippines
d. Non-resident alien not engaged in trade or business in
the
Philippines

53. In which of the following cases will the dividend income


from a foreign corporation be classified as “income without”
a. Less than 50% of the foreign company’s gross income for
the preceding three (3) years prior to the dividend
declaration was derived from sources within the Philippines.
b. 50% of the foreign company’s gross income for the
preceding three (3) years prior to the dividend
declaration was derived from sources within the
Philippines.
c. More than 50% of the foreign company’s gross income for
the preceding three (3) years prior to the dividend
declaration was derived from sources within the Philippines.
d. Always classified as income without”.

54. D’ Lion, Inc., a Philippine corporation, sold through the


local stock exchange 10,000 PLDT shares that it bought 2 years
ago. D’ Lion sold the shares for P2 million and realized a
net gain of P200,000. How shall it pay tax on the transaction?
a. It shall declare a P2 million gross income in its income
tax return, deducting its cost of acquisition as an
expense.
b. It shall report the P200,000 in its corporate income
tax return adjusted by the holding period.
c. It shall pay 5% tax on the first P100,000 of the
P200,000 and 10% tax on the remaining P100,000.
d. It shall pay a tax of one-half of 1% of the P2 million
gross
sales.

55. The payor of passive income subject to final tax is required


to withhold the tax from the payment due the
recipient. The withholding of the tax has the effect of
a. A final settlement of the tax liability on the income.
b. A credit from the recipient's income tax
liability. c. Consummating the transaction resulting in
an income. d. A deduction in the recipient's income tax
return.

56. Winterfell, Inc., bought a parcel of land in 2015 for P7


million as part of its inventory of real properties. In 2017,
it sold the land for P12 million which was its zonal
valuation. In the same year, it incurred a loss of P6 million
for selling another parcel of land in its inventory. These were
the only transactions it had in its real estate business.
Which of the following is the applicable tax treatment?
a. Winterfell shall be subject to a tax of 6% of P12 million.
b. Winterfell could deduct its P6 million loss from its P5
million gain.
c. Winterfell’s gain of P5 million shall be subject to
the holding period.
d. Winterfell's P6 million loss could not be deducted from
its
P5 million gain.
57. Passive income includes income derived from an activity in
which the earner does not have any substantial participation.
This type of income is
a. Usually subject to a final tax.
b. Exempt from income taxation.
c. Taxable only if earned by a
citizen. d. Included in the income tax
return.

58. In 2017, Alice earned P500,000 as income from her beauty


parlor and received P250,000 as Christmas gift from her aunt.
She had no other receipts for the year. She spent P150,000 for
the operation of her beauty parlor. For tax purposes, her gross
income for 2017 is
a. P750,000 c. P350,000
b. P500,000 d. P600,000

59. Which of the following items is not part of gross income to


be reported in the income tax return?
a. Increase in value of land
b. Gambling winnings
c. Prize of P10,000
d. Gain from sale of store’s air conditioner

60. Mr. Yu leased his lot to Mr. Uy. The contract calls for Mr. Uy
to construct a house which would serve as the residence
of the latter, the ownership thereof to be vested in Mr. Yu
after the expiration of the lease. When the
house was completely constructed, the remaining term of the lease
was 10 years. The residential house had an estimated useful life of
15 years.
What is the tax implication of the leasehold improvement?
a. Mr. Yu derives taxable income on the improvement; Mr. Uy
can claim depreciation expense as a deduction from gross
income.
b. Mr. Yu derives taxable income on the improvement; Mr.
Uy
cannot claim depreciation expense as a deduction from
gross income.
c. Mr. Yu does not derive taxable income on the
improvement; Mr. Uy cannot claim depreciation expense as a
deduction from gross income.
d. Mr. Yu does not derive taxable income on the
improvement; Mr. Uy can claim depreciation expense as a
deduction from gross income.

61. Assume the same facts in the immediately preceding number,


except that at the time of the completion of the residential
house, the remaining term of the lease was 15 years while the
useful life of the house was 10 years.
What is the tax implication of the leasehold improvement?
a. Mr. Yu derives taxable income on the improvement; Mr. Uy
can claim depreciation expense as a deduction from gross
income.
b. Mr. Yu derives taxable income on the improvement; Mr.
Uy cannot claim depreciation expense as a deduction from gross
income.
c. Mr. Yu does not derive taxable income on the
improvement; Mr. Uy cannot claim depreciation expense as a
deduction from gross income.
d. Mr. Yu does not derive taxable income on the
improvement; Mr. Uy can claim depreciation expense as a
deduction from gross income.

62. Which of the following expenses may be deducted from


gross compensation income?
a. Depreciation of permanent assets
b. Premium payments on health and/or hospitalization insurance
c. Bad debts written off
d. Optional standard deduction

63. Which of the following items of interest expense may be


deducted from gross income?
a. Interest on corporation’s preferred stock
b. Interest on loan for construction of a rest house
c. Interest for delinquency in the payment of percentage tax
d. Interest on bank loan to finance petroleum exploration

64. Which of the following taxes may be deducted from gross income?
a. Percentage tax on sale of listed stock
b. Business permit fee paid to the city government
c. Income tax
d. Tax on interest on bank deposit

65. Who among the following taxpayers may not claim a tax credit
or deduction on income tax paid to foreign countries?
a. Resident citizens
b. Resident aliens
c. Domestic corporations
d. General Co-Partnerships

66. The loss from sale or exchange of property is deductible


from gross income where the sale or exchange is:
a. Between fiduciary of a trust and the fiduciary of
another trust if they have the same grantor
b. Between fiduciary of a trust and the beneficiary of
such trust
c. Between an individual and his first cousin
d. Between an individual and a corporation if the former
owns more than 50% in value of the outstanding capital
stock of the latter

67. Anne, claimed a bad debt of P50,000 as a deductible expense in


the taxable year 2016. In 2017, Anne was able to recover the
P50,000 debt already written off in the preceding year.
What is the treatment for tax purposes of the recovery of the
bad
debt?
a. Report the recovery of the bad debt as gross income in
2016. b. Report the recovery of the bad debt as gross income in
2017.
c. Disregard the recovery of the bad debt.
d. Amend the 2016 income tax return to rectify the
deduction for bad debt claimed.

68. Which of the following assets shall be subject to


depletion?
a. Machinery
b. Land containing ore deposit
c. Commercial
d. Goodwill

69. Which of the following individual taxpayers may claim basic


and additional personal exemptions for income tax purposes?
a. Non-resident aliens engaged in trade or business in the
Philippines, in the absence of reciprocity
b. Non-resident aliens not engaged in trade or business in
the
Philippines
c. Both (a) and (b)
d. Neither (a) nor (b)

70. Harry works as financial consultant in an oil firm in Dubai.


Aside from his salary thereat, he also maintains a 10-door
apartment in Manila which he inherited from his parents when
he was already married. On the other hand, Wilma, his wife, is
employed as a loan officer at a local bank. Data
pertaining to their dependents appear below for the taxable
year 2017:

 Anton - Son who turned 23 on April 1, 2017; incapable


of self-support due to loss of both legs in an accident;
 Bunny - 21 year old daughter who is taking up culinary
arts in Paris, France;
 Charlie - 15-year old adulterous son of Harry living with
the couple;
 Dina - 12-year old child who died from a vehicular
accident on January 1, 2017; and
 Evan - 80-year old father of Wilma, supported by her
and living with the couple.

The basic and additional personal exemptions of Harry for the


taxable year 2017 amounts to:
a. P50,000 and P100,000, respectively
b. P50,000 and P75,000,
respectively c. P50,000 and P0,
respectively
d. P0 and P0, respectively

71. Assume the same facts above, the basic and additional
personal exemptions of Wilma for the taxable year 2016 amount to:
a. P50,000 and P100,000,
respectively b. P50,000 and P75,000,
respectively c. P50,000 and P0,
respectively
d. P0 and P0, respectively
72. Which of the following statements does not characterize a
capital asset?
a. It may be real or personal property.
b. It is not always subject to a holding period.
c. It is normally subject to value-added tax when it is sold.
d. It is not always subject to a final tax.

73. Which of the following transactions is exempt from capital


gains tax?
a. The sale of the principal residence of the taxpayer
where the entire proceeds is used to purchase a vacation
lot at Tagaytay
b. The sale of a beach lot of the taxpayer where the
entire proceeds is used to construct his principal
residence
c. The sale of the principal residence of the taxpayer for
the second time in ten (10) years to purchase another principal
residence
d. None of the choices

74. Which of the following transactions is treated as a capital


asset transaction for income tax purposes?
a. Sale of a residential lot by a subdivision
developer b. Sale of a used delivery truck by
a retailing company c. Liquidation of partnership business
d. Sale of shares of stock by a dealer in securities

75. Which of the following is not an attribute of a deferred-


payment sale?
a. The initial payments exceed 25% of the selling price in
the year of sale.
b. The obligations or promissory notes received by the
vendor from the vendee are considered as equivalent to
cash.
c. The tax may be paid in installments.
d. The sale involves both real and personal property.

76. The deductible expenses of an estate may consist


of:
a. Deductible expenses allowed to an individual
taxpayer b. Income distributed to beneficiaries
c. Both (a) and (b)
d. Neither (a) nor (b)

77. Determine which of the following trusts shall the taxable


income be consolidated and the income tax thereon computed on
the basis of such consolidated income?
a. Trust No. 1 and Trust No. 2 have the same grantor and
with different beneficiaries.
b. Trust No. 1 and Trust No. 2 have the same grantor and
the
same beneficiary.
c. Trust No. 1 and Trust No. 2 have different grantors and
the same beneficiary.
d. Trust No. 1 and Trust No. 2 have different grantors and
the same fiduciary and beneficiary.
78. Inday is a resident citizen of the Philippines. Data for a
year: Gross income from business P 700,000
Royalty from books 40,000
Gain on direct sale to buyer of
shares of stock of a domestic corporation
held as capital asset 70,000
Loss on sale of land in the Philippines held
as capital asset with cost of P1,500,000 when the
zonal value is P1,200,000 500,000
Business Expenses 300,000
How much is the total income tax expense for the year?
a. P177,500 c. P159,500
b. P80,000 d. P156,000

Gross Income from Business 700,000


Less: Business Expense 300,000
Personal Exemptions 50,000 350,000
Net Taxable income 350,000

Tabular Schedule:
1st 250,000 Tax is 50,000
Excess of 250,000 - 350,000 = 100,000 x 30% 30,000
Capital Gains Tax on Shares (70,000 x 5%) 3,500
Royalty Income (40,000 x 10%) 4,000
Loss
Total on Sale Tax
(Zonal Value 1.2 x 6%) 72,000
Income Expense 159,500

79. Mercy is a citizen and of


resident of Philippines. She ahad
compensation income (net exclusions) of P200,000 and net
income from business of P700,000 for a year. She made
quarterly
income tax payments amounting to P237,000 and her employer
withheld P25,000 on her compensation income.
The income tax payable (refundable) for the year
is: a. (P25,000)
c. (P42,000) b.
P237,000 d. (P37,500)

Compensation Income 200,000


Net Income from Business 700,000
Total Income 900,000
Basic Exemptions (50,000)
Taxable Income 850,000
1st 500,000 Tax 125,000
Excess of 500,000- 850,000 x 32% 112,000
Total Tax Due 237,000
Less: Taxes Payments
Quarterly Payments 237,000
Compensation CWT 25,000 262,000
Refundable /Creditable Tax (25,000)
80. EMT has the following data on his passive income earned during
the year 2016:
Philippines Abroad
Interest income from bank deposits 45,000 25,000
Interest income from FCDU 50,000 -0-
Royalties 20,000 30,000
Royalties from
from books
computer programs 20,000 40,000
Dividend income from a domestic corporation 27,000 13,000
Dividend income from a foreign corporation 33,000 22,000

How much is the final withholding tax if the taxpayer is a


resident citizen?
a. P21,450 c. P17,700
b. P20,400 d. P36,250

Philippines
Interest income from bank deposits 45,000 x 20% = 9,000
Interest income from FCDU 50,000 x 7.5% = 3,750
Royalties from books 20,000 x 10% = 2,000
Royalties from computer programs 20,000 x 20% = 4,000
Dividend income from a
domestic corporation 27,000 x 10% = 2,700
Total Withholding Taxes 21,450

81. How much is the final withholding tax if the taxpayer is a


resident alien?
a. P21,450 c. P17,700
b. P20,400 d. P36,250
SAME COMPUTATION
AS RESIDENT CITIZEN

82. How much is the final withholding tax if the taxpayer is a


non- resident citizen?
a. P21,450 c. P17,700
b. P20,400 d. P36,250
Philippines
Interest income from bank deposits 45,000 x 20% = 9,000
Royalties from books 20,000 x 10% = 2,000
Royalties from computer programs 20,000 x 20% = 4,000
Dividend income from a domestic
corporation 27,000 x 10% = 2,700
Total Withholding Taxes 17,700

83. How much is the final withholding tax if the taxpayer is a


non- resident alien engaged in trade or business?
a. P21,450 c. P17,700
b. P20,400 d. P36,250
Philippines
Interest income from bank deposits 45,000 x 20% = 9,000
Royalties from books 20,000 x 10% = 2,000
Royalties from computer programs 20,000 x 20% = 4,000
Dividend income from a domestic
corporation 27,000 x 20% = 5,400
Total Withholding Taxes 20,400

84. How much is the final withholding tax if the taxpayer is a


non- resident alien not engaged in trade or business?
a. P21,450 c. P17,700
b. P20,400 d. P36,250

Interest income from bank deposits 45,000


Royalties from books 20,000
Royalties from computer programs 20,000
Dividend income from a domestic corporation 27,000
Dividend income from a foreign corporation 33,000
TOTAL PASSIVE INCOME 145,000
Final Withholding Tax Rate 25%
Total Withholding Taxes 36,250
85. Nonoy is an employee of a firm in Quezon City. He is
supporting
his 4 year income
compensation old brother whoyear
for the living with him.
is shows: Data on his
Regular Salary P 240,000
Thirteenth month pay 20,000
Quarterly bonus 40,000
Payroll Deductions:
SSS Premiums 3,000
Philhealth contributions 1,200
Pagibig contributions 4,000
Labor union dues 1,000
Premium payments on insurance 3,000
hospitalization
Payment of loan 5,000
86. How much is the taxable income?
a. P210,800
b. P207,800 c.
d. P208,400
P260,800

Regular Salary P 240,000


Total Income 240,000
Less: Basic Exemption (50,000)
SSS Premiums (3,000)
Philhealth contributions (1,200)
Pagibig contributions (4,000)
Labor union dues (1,000)
Net Taxable Income 180,800 +30,000 if old laws = 218,000
87. A domestic corporation, in its fifth (5th) year of operations,
had the following data for the year:
Net sales P 2,000,000

Capital gain on direct sale to a buyer


of shares of a domestic corporation for P500,000 200,000

Capital gain on sale thru a real


estate broker of land and building outside
the Philippines for P5,000,000 1,000,000

Dividend corporation 50,000


Interest from a
on bank 40,000
deposit
Cost of sales 600,000
Quarterly corporate income tax paid 190,000
Operating expenses 500,000
The income tax payable upon filing of the annual income tax
return
is:
a.
b. P425,000
P380,000 c.
d. P570,800
P520,800

Net Sales 2,000,000


Cost of Sales (600,000)
Operating Expenses (500,000)
Net Income from Ordinary Business 900,000
Add Other Income Broker Outside Phils 1,000,000
Total Taxable Income 1,900,000
Corporate Tax Rate 30%
Income Tax Due 570,000
Less Quarterly Income Tax Paid (190,000)
Income Tax Due and Payable 380,000

88. The Kultura Foundation of the Philippines, a non-stock, non-


profit corporation, organized and operated exclusively to
preserve and show-case Philippine cultural practices, music,
dances, and folk arts, deriving funding from mostly donations,
had the following data for the year:

Donations received P 20,000,000


Interest income from bank deposit 100,000
Rent income from properties received as donation
(net of 5% withholding tax) 475,000
Expenses related to its rent income 30,000

89. How the income tax expense of the Corporation


year?much is for
P161,000
a. c.
b. P136,000 d. P172,000
P125,600

Rent Income (Gross Up) 500,000


Less: Expenses (30,000)
Net Rent Income 470,000
Tax Rate 30%
Tax Expense 141,000
Add Interest Income Tax 20,000
Total Income Tax Expense 161,000

90. A tax imposed in the nature of a penalty to the corporation


to deter tax avoidance of shareholders who avoid paying the
dividends tax on the earnings distributed to them by the
corporation.
a. Minimum corporate income tax
b. Optional corporate income
tax
c. Improperly accumulated earnings tax
d. Capital gains tax

91. First statement: An accumulation of earnings or profits


(including undistributed earnings or profits of prior years) is
unreasonable if it is not necessary for the purpose
of the business, considering all the circumstances of the
case.
Second statement: The term "reasonable needs of the business"
are
hereby construed to mean the immediate needs of the business,
including reasonably anticipated needs.
a. Only the first statement is correct
b. Only the second statement is
correct c. Both statements are correct
d. Both statements are incorrect

92. The Improperly Accumulated Earnings Tax (IAET) is imposed


on improperly accumulated taxable income earned starting
January 1,
1998 by domestic corporations as defined under the Tax Code
and which are classified as closely-held corporations at the rate
of:
a. twenty percent (20%). c. ten percent (10%).
b. fifteen percent (15%). d. five percent (5%).

93. The Improperly Accumulated Earnings Tax shall not apply to


which of the following corporations?
a. Banks and other non-bank financial
intermediaries b. Insurance companies
c. Publicly-held corporation
d. All of the choices

94. First statement: Once earnings


the profits havesame
been subjected to
improperly accumulated tax, the shall no be
longer as
subject to the same tax in later years even if not
declaredstatement: Profits which have been subjected to
Second
improperly
accumulated earnings tax when finally declared as dividends
shall be subject to tax on dividends.
a. Both statements are true
b. Both statements are false
c. Only the first statement is true
d. Only the second statement is true
95. To avoid payment of IAET, when must the dividends be declared
and paid or issued?
a. Not later than one year following the close of taxable
the
year
b. Not later than the 15th day following the close of the
taxable year
c. Not later than 60th day following the close of taxable
the
quarter
d. None of the choices

TAX REMEDIES
96. Rosalie, a compensation income earner, filed her income tax
return for the taxable year 2013 on March 30, 2014. On May
20, 2017, Rosalie received an assessment notice and
letter of demand covering the taxable year 2013 but the
postmark on the envelope shows April 10, 2017. Her return is
not a false and fraudulent return. Can Rosalie raise the defense
of prescription?
a. No. The 3 year prescriptive period started to run on
April
15, 2014, hence, it has not yet expired on April 10, 2017.
b. Yes. The 3 year prescriptive period started to run on
April
15, 2014, hence, it had already expired by May 20, 2017.
c. No. The prescriptive period started to run on March 30,
2014, hence, the 3 year period expired on April 10, 2017.
d. Yes. Since the 3-year prescriptive period started to run
on
March 30, 2014, it already expired by May 20, 2017.

97. On March 30, 2012 Emmett Foods, Inc. received a notice


of assessment and a letter of demand on its April 15, 2009 final
adjustment return from the BIR. Emmett Foods then filed a
request for reinvestigation together with the requisite
supporting documents on April 25, 2012. On June 2, 2012, the
BIR issued a final assessment reducing the amount of the tax
demanded. Emmett Foods was satisfied with the reduction, it
did not do anything anymore. On April 15, 2017 the BIR
garnished the corporation's bank deposits to answer for the tax
liability. Was the BIR action proper?
a. Yes. The BIR has 5 years from the filing of the
protest within which to collect.
b. Yes. The BIR has 5 years from the issuance of the
final
assessment within which to collect.
c. No. The taxpayer did not apply for a compromise.
d. No. Without the taxpayer’s prior authority, the BIR
action
violated the Bank Deposit Secrecy Law.

98. Renesmee, Inc. received a notice of assessment and a letter


from the BIR demanding the payment of P3 million pesos in
deficiency income taxes for the taxable year 2015. The
financial statements of the company show that it has been
suffering financial reverses from the year 2016 up to the
present. Its asset position shows that it could pay only
P500,000.00 which it offered as a compromise to the BIR.
Which among the following may the BIR
require to enable it to enter into a compromise with
Renesmee, Inc.?
a. Renesmee must show it has faithfully paid taxes before
2016.
b. Renesmee must promise to pay its deficiency when
financially able.
c. Renesmee must waive its right to the secrecy of its
bank
deposits.
d. Renesmee must immediately deposit the P500,000 with the
BIR.

99. As a rule, within what period must a taxpayer elevate to the


Court of Tax Appeals a denial of his application for refund
of income tax overpayment?
a. Within 30 days from receipt of the Commissioner’s denial
of his application for refund.
b. Within 30 days from receipt of the denial which must
not
exceed 2 years from payment of income tax.
c. Within 2 years from payment of the income taxes sought to
be refunded.
d. Within 30 days from receipt of the denial or within
two years from payment.

100. What is the effect on the tax liability of a taxpayer who does
not protest an assessment for deficiency taxes?

a. The taxpayer may appeal his liability to the CTA since


the assessment is a final decision of the Commissioner
on the matter.
b. The BIR could already enforce the collection of
the taxpayer's liability if it could secure authority
from the CTA.
c. The taxpayer's liability becomes fixed and subject to
collection as the assessment becomes final and collectible.
d. The taxpayer's liability remains suspended for 180 days
from the expiration of the period to protest.

101. The taxpayer seasonably filed his protest together with all
the supporting documents. It is already July 31, 2017, or
180 days from submission of the protest but the BIR
Commissioner has not yet decided his protest. Desirous of an
early resolution of his protested assessment, the taxpayer
should file his appeal to the Court of Tax Appeals not later
than
a. August 31, 2017.
b. August 30, 2017.
c. August 15, 2017.
d. August 1, 2017.
DOCUMENTARY STAMP TAX
102. A newly formed corporation issued shares of stocks to
its incorporators for P150,000. The par value of the shares issued is
P100,000. How much is the documentary stamp tax?
a. P500 c. P750
b. P1,000 d. P1,500

Par Value Divide by 200 x 1 peso = 100,000 / 200 = 500

103. Mr. T invested in shares of stock of Kapisananngmga Sisters


Inc. amounting to P100,000 with par value of P80,000. After 2 years, he
disposed said shares directly to Mr. B for P230,000.
The documentary stamp tax on above transaction is:
a. P400 c. P300
b. P1,150 d. P862.50

**80,000 / 200 = 400 x 75% = 300

104. Based on number 97, but assuming the shares are without par
value, how much is the documentary stamp tax?
a. P500 c. P750
b. P1,000 d. P0.00

**Selling Price - Cost = 230,000 - 80,000 = 150,000 / 200 = 750

105. Continuing number 99, if the shares were subsequently sold


for
P200,000, how much is the documentary stamp tax?
a. P750 c. P1,500
b. P187.50 d. P375

**Doc Stamp Upon Original Issuance 750 x 25% = 187.50

106. Mr. Purisima owns a resthouse in Pampanga acquired by him


for P10,750,000. He sold the same to Mr. Apo for
P5,000,000. The fair market value at the time of sale per
assessor’s office is P10,000,000 while zonal value is
P15,000,000. The documentary
stamp tax on the transaction is:
a. P75,000 c. P150,000
b. P225,000 d. P161,250

**15,000,000/1000 = 15,000 x 15 pesos =


225,000

107. Who is liable to the payment documentary stamp tax?


a. Mr. Purisima
b. Mr. Apo
c. It depends on the agreement of the parties
d. It depends on who is benefiting on the transaction
TAXATION QUIZZER PART 2

1. The BIR is assessing deficiency withholding tax on total Repairs


and Maintenance claimed by non-stock non-profit organization. The
BIR alleges that foundation failed to withhold the correct amount
of the
2% expanded withholding tax due on the Repairs
and Maintenance expense. As a CPA assisting the foundation in the BIR tax
investigation, you can reason that .
a. The BIR should have considered the 1% expanded withholding
tax instead of the 2%in determining the deficiency tax.
b. The foundation is exempt from the tax and hence is not liable
to
the deficiency withholding tax.
c. The foundation is exempt from tax and hence is not
required to
withhold tax on its expenses.
d. The BIR should have considered both the 1% and 2% withholding
tax in determining the deficiency tax.
c

person, property, or
excises. d. It is generally in
money

4. Which of the statement below is grammatically correct?


a. The imposition of minimum corporate income tax may be
suspended
if substantial losses are sustained due to a prolonged labor
dispute.
b. The imposition of minimum corporate income tax maybe suspend
if substantial losses are sustained due to prolonged labor
dispute.
c. The imposition of minimum corporate income tax may be
suspended if substantial losses are sustained due to a prolong
disputes.
d. The imposition of minimum corporate income tax may be
suspended.
If substantial loss are sustained due to a prolonged labor
dispute.

c
6. The BIR issued a tax assessment against the taxpayer who was not
given sufficient time to protest the said assessment. The
taxpayer noted that their competitors were issued tax
assessments but were given
enough time to protest. The BIR violated the of the
constitution.
a. Equal protection clause c. Equitability principle
b. Due process clause d. Uniformity principle

Items 7-10 are based on the following information:

On March 30, 2012, XXX, Inc., received a notice of assessment and a


letter of demand on its April 15, 2009 final adjustment return from
the BIR. XXX, Inc., then filed a request for reinvestigation together with
the requisite supporting documents on April 25, 2012. On June 2,
2012, the BIR issued a final assessment reducing the amount of the
tax demand. Since XXX, Inc., was satisfied with the reduction,
it did not do anything anymore. On April 15, 2017 the BIR
garnished the corporation’s bank deposits to answer the liability.

7. Was the BIR action proper?


a. No, the taxpayer did not apply for the compromise.
b. Yes, the BIR has 5years from the filing within which to
collect. c. No, without the taxpayer’s prior
authority, the BIR action
violated the Bank Deposit Secrecy Law.
d. Yes, the BIR has 5 years from the issuance if the final
assessment within which to collect.

8. What is the effect of the XXX, Inc.’s failure to file a protest on


its assessed deficiency taxes?
a. The taxpayer may file a motion for reconsideration to the CIR
on the matter.
b. The taxpayer may appeal his liability to the CTA since the
assessment is a final decision of the Commissioner on the
matter.
c. The taxpayer’s liability becomes fixed and subject to
collection
as the assessment becomes final and collectible.
d. The BIR could already enforced the collection of the
taxpayer’s liability if it could secure authority from the CTA.

9. Which statement is correct? The collection of a deficient


tax assessment by distraint and levy:
a. May be done only once during the taxable year.
b. Must be done successively, first by distrait and then by levy;
c. May be repeated, if necessary, until the full amount due,
including all expenses, is collected;
d. Automatically covers the bank deposits of a delinquent taxpayer.

c
c. Tax assessment refers to the process of determining the
correct amount of tax due in accordance with the prevailed tax laws.
d. Tax assessment refers to the process of determining the
correct
amount of tax due in accordance with the prevailing tax
laws.

11. A joker was commissioned in a kiddy party to perform magic. The


comic was to be paid P100, 000 for his performance and the
parties signed the necessary contract. He then gratuitously assigned
his rights under the contract to his son. The son later on
collected the P100,
000 talent fee of his father from the contractee. The national
internal revenue tax/es payable is/are:
a. Income tax only.
b. Donor’s tax only.
c. Both income and donor’s taxes.
d. Neither income tax nor donor’s tax.

13. Amount receivable or


administration as by the estate under
a beneficiary of the deceased,
life hispolicy
insurance executor by
takendecedent upon his own life is:
the
a. Excluded from gross state.
b. Part of gross state if the beneficiary is revocable.
c. Excluded from gross state if the beneficiary is irrevocable.
d. Part of gross state whether the beneficiary is revocable
or
irrevocable.

14. The following journal was made


VAT-registered taxpayer:entry in the purchases journal of

Purchases xxx
Cash or Accounts Payable xxx

The journal entry signifies that:


a. Purchases were from a non-VAT supplier.
b. Purchases were from a VAT-registered supplier.
c. Purchases were either from non-VAT or VAT-registered
supplier. d. Input tax has been taken as part of the cost of
purchase.

15. A common carrier by land is engaged in the transport of


passengers, goods and cargoes. He is not VAT-registered. What
business tax or taxes is he liable to pay?
a. 12% value-added tax
b. 3% common carrier’s tax
c. 12% VAT on gross receipts from transport of goods and cargoes
and
3% common carrier’s tax on gross receipts from transport of
passengers.
d. 3% tax on VAT-exempt persons on gross receipts from transport
of
passengers.

16. Statement 1. As to the property of the state, exemption is the


rule and taxation the exception.
Statement 2. As of the property of the taxpayer, taxation is the rule
and exemption the exception.

a. Both statements are correct


b. Both statements are incorrect.
c. Statement 1 is correct while statement 2 is
incorrect. d. Statement 1 is incorrect while statement 2
is correct.

17. Foreign income taxes paid by the resident citizen or


domestic corporation.
a. May be claimed only as tax credit.
b. May be claimed only as tax deduction.
c. Do not qualify either as a tax credit or as a tax deduction.
d. May be claimed either as a tax credit or as a tax deduction
at
the option of the income taxpayer.

18. Statement 1. Government agencies performing essential


governmental functions are subject to tax unless expressly exempted.
Statement 2. Government agencies performing propriety function are
exempt from tax unless expressly taxed.

a. Both statements are correct.


b. Both statements are incorrect.
c. Statement 1 is true while statement 2 is
false. d. Statement 2 is true while statement 1 is
false.

19. Statement 1. Tax avoidance or tax minimization is the use by


the taxpayer of legally permissible methods in order to
reduce tax liability.
Statement 2. Tax evasion or tax dodging is the use by the taxpayer
of illegal means to defeat or lessen the payment of tax.

a. Both statements are correct.


b. Both statements are incorrect.
c. Statement 1 is correct while statement 2 is
incorrect. d. Statement 2 is correct while statement 1
is incorrect.

20. Which one among the following items below is included in the
gross state?
a. Revocable transfer.
b. Transfer with reservation of certain rights.
c. Transfer under general power of appointment.
d. Transfer in contemplation of death which is onerous.
21. Which of the following statement is
correct?
a. The final tax on compensation of special kind of non-
resident aliens is 25% of the gross income.
b. Interest income from a foreign currency deposit unit in the
Philippines of a non resident alien is not subject to final tax.
c. Informer’s reward is subject to final tax of 10% based on the
10%
of the value of tax assessed or P1, 000,000 whichever is
higher. d. Prizes exceeding P10,000 derived by non-resident alien not
engage
in trade or business here in the Philippines is subject to a
final tax of 20%

22. The power to decide disputed assignment, refunds of internal


revenue taxes, fees or other charges, penalties imposed in relation
thereof, or other matters arising other the tax code or other laws
or portions thereof administered by the BIR is vested in the:
a. Secretary of Finance
b. Commissioner of Internal Revenue
c. Court of Tax
Appeals d. Regional Trial
Court

23. The commissioner of the Internal Revenue (CIR) is prohibited by law


to look into the bank accounts of taxpayer, except when:
a. Taxpayer is accused of heinous crime
b. Taxpayer did not invoke his right to privacy during the tax
audit.
c. Taxpayer applies for compromise on tax obligation on account
of
financial incapacity.
d. The CIR has reason to believe that taxpayer has filed a false
or fraudulent return.

24. A VAT-registered realty company sells real property in the course


of its business. On April 30, 2016, it has sold a lot under the
following items (VAT excluded).

Selling Price P4,000,000


Down payment, 4/30/16 400,000
1st Installment, 4/30/17 600,000
2nd Installment, 4/30/18 1,000,000
3rd Installment, 4/30/19 1,000,000
4th Installment, 4/30/20 1,000,000
Interest and other charges of
10% On unpaid balance per
installment

The zonal value of the lot at the time of sale is P4, 800,000.

The output VAT for the installment received on April 30, 2017, is:
a. P43,200
b. P86,400
c. P115,200
d. P129,600
Output Tax (4,800,000 x 12%) = 576,000
Principal 600,000 / 4,000,000 x 576,000 = 86,400
Interest 3,600, 000 x 10% x 12% = 129,600
Total

25. Counting No. 24, the output VAT on April 30, 2020, is:
a. P132, 000
b. P144,000
c. P156,000
d. P228,000

4,800,000 x 12% = 576,000 out put tax


Principal 1,000,000 / 4,000,000 x 576,000 = 144,000
Interest 1,000,000 x 10% x 12% = 12,000
Total 156,000

26. Assuming that the real property in No. 24 is the residential lot
sold for cash of P1, 750,000 (VAT not separately blend in
the sales document) on April 30, 2016. Zonal value of the lot at
the time of sale is P1, 900,000. The output VAT on the sale is:
a. P187,500
b. P210, 000
c. P228,000
d. None, as it is exempt the VAT.

27. The lease of real or personal property is subject to:


i. 7% gross receipt tax (GRT) if the lessor is the bank.
ii. 3% percentage tax or 12% value-added tax (VAT) if the
lessor is not a bank.

a. No to I and II
b. Yes to I and
II c. Yes to I
only
d. Yes to II only

28. Ayala Land, Inc. (ALI) bought a parcel of land in 2014 for P7
million as part of its inventory of real properties. In 2016, it
sold the land for P12 million which was its zonal valuation. In
the same year, it suffered a loss of P6 million for selling another
parcel of land from its inventory. These were the only transactions
ALI had in its real estate business. Which of the following
is the applicable tax treatment?
a. ALI shall be subject to a tax of 6% of P12 million.
b. ALI’s gain of P5 million shall be subject to holding period.
c. ALI could deduct its P6 million loss from its P5 million gain.
d. ALI’s P6 million loss could not be deducted from its P5
million gain.

29. This is an inherent limitation on the power of taxation:


a. The rule of taxation shall be uniform and equitable.
b. No law impairing the obligations of contacts shall be enacted.
c. Tax laws can not apply to the property of foreign governments.
d. Charitable institutions, churches, parsonages, convents and
all lands, buildings and improvements, actually, directly,
and exclusively used for religious, charitable or educational
purposes shall be exempt for taxation.

30. X took a life insurance policy of P5 million where the


monthly premium is P10, 000. The proceeds will be paid to X
after 25 years to the X’s estate should X die before completing
the equivalent of
25 years payment. If the X outlived the policy, which of the
following is correct?
a. The proceeds will be part of X’s gross estate.
b. The proceeds will be part of X’s taxable income.
c. The proceeds will be party taxable estate and partly
exempt. d. The proceeds will be party taxable income and
partly exempt.

31. Containing the preceding number, except that after paying


the equivalent of ten years premium, X transferred the policy to
Y for P1.5 million and Y continued paying the monthly premium
as they mature. After 10 years, X died. Which of the following is
correct?
a. The proceeds will be part of X’s gross estate.
b. The proceeds received by Y is part of his taxable income.
c. The amount received by X from Y is part of X’s taxable income.
d. The amount received by the X from Y and the proceeds received by
Y
are partly taxable income and partly
exempt.

32. An owner of several warehouses for rent, which used to be


VAT- exempt because its annual gross receipts never exceeded
P1,919,500 decided to register under the VAT system on January
2, 2016. The following data were from the first quarter ending
March 31, 2016:

Rental from warehousing services, net of VAT P


672,000
Purchases of supplies, gross of VAT
224,000
Inventory of supplies, January 1, 2016
201,600
Actual VAT paid on the inventory
of
supplies, January 1, 2016
21,600

The value added tax payable for the quarter


is:
a. P23,520
b. P35,040
c. P41,088
d. P52,608

VAT on rental (672,000 x 12 %) 80,640


VAT on purchase of supplies (224,000 x 12/112) (24,000)
Transitional Input Tax(higher) (21,600)
VAT Payable 35,040
33. Assuming that one vacant warehouse in the preceding number
was rented out for the whole month of April 2016 and received
P107, 000 as rent, gross of VAT but net of the
applicable creditable withholding income tax (expanded). The
output VAT on the rental receipt is:
a. P12,000
b. P12,240
c. P12,840
d. P13,440

107,000/107% = 100,000 x 12% =


12,000

Numbers 34 to 42 are based on the following


information:

Dina Cabangon, a citizen and resident of the Philippines, died on


November
1, 2016. Her marriage was under the system of absolute community
of property. The following properties and obligations were left:

Property received by Dina as inheritance on February 1,


2015
(during the marriage)
P2,000,000
Real property acquired through the labor of both Dina
and
her Husband during the marriage (family home)
4,000,000
Property owned by Dina before marriage
300,000
Property owned by Dina’s Husband before marriage
200,000
Funeral expenses
300,000
Unpaid mortgage on property inherited
200,000
Judicial expenses for the settlement of the estate
80,000
Unpaid obligations (excluding the unpaid mortgage)
40,000

The property received as inheritance was part of the gross estate of


the prior decedent at a fair market value of P1,100,000. At the
time of inheritance, it was mortgage for P300,000. Dina was able to
pay P100,000 before she passed away.

34. The total community property is:


a. P6,500,000 c. P4,000,000
b. P4,500,000 d. P2,300,000

35. The total exclusive property is:


a. P4,200,000 c. P2,300,000
b. P2,500,000 d. P2,000,000
36. The total ordinary community deduction is:
a. P200,000 c. P420,000
b. P320,000 d. P445,000

37. The total is:


ordinary exclusive deduction (excluding vanishing
deduction)
a. P400,000
b. P300,000 c.
d. P200,000
P100,000
38. The deduction for family home is:
a. P0 c. P2,000,000
b. P1,000,000 d. P4,000,000

39. The amount of vanishing deduction is:


a. P0 c. P736,000
b. P816,000 d. P656,000

40. The total special deduction is:


a. P0 c.P2,000,000
b. P1,000,000 d.P4,000,000

41. The taxable net estate is:


a. P1,154,000 c. P3,154,000
b. P2,154,000 d. P4,244,000

SOLUTION: Separate Common Total


2,000,000 4,000,000
300,000
200,000
Gross Estate 2,000,000 4,500,000 6,500,000
Funeral Expenses:
Actual 300,000
Limit (6.5 x 5%) 325,000
Threshold 200,000
Whichever is lower (200,000)
Judicial Expenses (80,000)
Indebtedness (200,000) (40,000)
Vanishing Deductions* (736,000)
Net Estate After
Ordinary Deductions 1,064,000 4,180,000 5,244,000
Family Home (1,000,000)
Standard Deductions (1,000,000)
Share of Surviving 1/2 x 4,180,000 (2,090,000)
Spouse 1,154,000
Taxable NE
*Vanishing Deduction:
Lower FMV 1,100,000
Mortgage Assumed and Paid (100,000)
Initial Basis 1,000,000
Pro Rated Deductions:
1,000,000/6,500,000 x 520,000 (80,000)
Final Basis 920,000
Vanishing Rate _ 80%
Vanishing Deductions 736,000

42. Going back to the original


Dina Cabangon to her spouseproblem, except
was under the that
systemtheof marriage
conjugal
partnership of gains. The taxable net estate would be:
a. P3,206,032 c. P1,206,032
b. P2,206,033 d. P1,154,000
Separate Common Total
2,000,000 4,000,000
300,000
Gross Estate 2,300,000 4,000,000 6,300,000
Funeral Expenses:
Actual 300,000
Limit (6.5M x 5%) 325,000
Threshold 200,000
Whichever is lower (200,000)
Judicial Expenses (80,000)
Indebtedness (200,000) (40,000)
Vanishing Deductions** (733,968)
Net Estate After
Ordinary Deductions 1,366,032 3,680,000 5,046,032
Family Home (1,000,000)
Standard Deductions (1,000,000)
Share of Surviving 1/2 x 4,180,000 (1,840,000)
Spouse 1,206,302
Taxable NE
**Vanishing Deduction:
Lower FMV 1,100,000
Mortgage Assumed and Paid (100,000)
Initial Basis 1,000,000
Pro Rated Deductions:
1,000,000/6,300,000 x 520,000 (82,540)
Final Basis 917,460
Vanishing Rate 80%
Vanishing Deductions 733,968

43. In March 2016, Imelda, who is fond of jewelries, bough


the following: diamond ring for P750, 000; bracelet for
P250, 000; necklace for P500, 000; and a brooch for P500, 000.
Imelda drives income from the exercise of her profession as a
topnotch Interior Designer. In October 2016. Imelda sold her
diamond ring, bracelet and necklace for only P1.25 million,
incurring a loss of P250, 000. She used the P1.25 million to
buy a solo diamond ring in November
2016 which she sold for P1.5 million in September 2017. Imelda
had no other transaction on jewelry in 2017. Which among the
following best describes the tax implications arising from the
aforesaid transactions?

a. Imelda may carry over and deduct her 2016 loss only from her
2017 gain.
b. Imelda may deduct her 2016 loss from both her 2017
professional income.
c. Imelda may not deduct her 2016 loss from both her 2017
professional income and her gain.
d. Imelda may carry over and deduct her 2016 loss from her 2017
professional income as well as from her
gain.
44. Taxation could be exercise by the following except one. Which one?
a. Judiciary
b. Legislative
c. Local government unit
d. President of the Philippines, in certain cases.

45. Stages, aspects or processes in


taxation. a. Levy of the tax
b. Collection of the tax.
c. Payment of the tax by the
taxpayer. d. All of the above.

46. Statement 1. Onerous donations are subject to donor’s tax.


Statement 2. Gratuitous donations are not subject to donor’s
tax. a. Both statement s are true
b. Both statements are false
c. Only statement 1 is true but not statement
2. d. Only statement 2 is true but not
statement 1.

47. Which of the following statements are correct?


a. Gift-splitting is a form of tax dodging.
b. The uncle who is the brother of the donor’s mother-in-law
is a non-stranger to the donor for purposes of the donor’s tax.
c. A gift made to a relative in January 2016 is to be added to
the gift made to the same relative in December 2015 in
determining the gift tax.
d. Renunciation by an heir including the surviving spouse of
his/her
share in the hereditary estate left by the decedent is subject
to donor’s tax if done in favor of identified heirs to the
exclusion or disadvantage of the other co-heir/s in the
hereditary estate.

48. Mistah, single and sales executive of a leading pharmaceutical


firm
(RiteMed), received in 2016 the following from his employer:

Salary, net of P267,000 withholding tax P 683,000


Allowances and benefits received:

 Rent paid by RiteMed on the house which Mistah occupies for


residential purposes, net of 5% withholding 129,200

 Entertainment allowance subject to liquidation (P75,000 was duly


receipted in the name of RiteMed and used to entertain RiteMed’s
customers and the balance of P25,000 was used to purchase a
late
model mobile phone for the personal use of Mistah) 100,000

 Reimbursement of entertainment expenses paid by Mistah


(P17,500 was used to entertain Mistah’s boyhood pals and the
balance of P22,500 was used to promote RiteMed’s
businesses.)
40,000
 Fixed yearly allowance for entertainment
85,000

The fringe benefit tax is:


a. P50,400
b. P52,000
c. P84,000
d. P92,000

Gross Rent on Housing 129,200/95% = 136,000 x 50% =


68,000
Expense Account (CP & Pal Exp) 25,000 + 17500
42,500
Total
110,500

110,500/68%
162,500
162,500 x 32%
52,000

49. Continuing number 48, the income tax payable by Mistah is:
a. P13,200 old
law
b. P27,760
c. P29,200
d. P43,360

Compensation(683,000 + 267,000)
950,000
Fixed Year Allowance
85,000
Personal Exemptions
(50,000)
Taxable Income
985,000

Income Tax per Tabular (OLD LAW)


up to 500,000 125,000
Excess (485,000 x 32%) 155,200
Withholding Tax (267,000)
Income Tax Payable 13,200

Income Tax per Tabular (TRAIN LAW)


up to 800,000 130,000
Excess (185,000 x 30%) 55,500
Withholding Tax (267,000)
Income Tax Payable (Refund) (81,500)

Numbers 50 to 56 are based on the following information:


Domestic Export
Sales to private entities P500,000 P500,000
Sales to the government 500,000
Sales of exempt goods 500,000
Input taxes passed on by VAT-registered suppliers on:
Sales to private entities 30,000 20,000
Sales to the government 25,000
Sales of exempt goods 10,000
Purchase of depreciable capital goods not
attributable to any specific activity (monthly
amortization for 60 months) 120,000
The sales to the government were subjected to the automatic deduction of
the
1% creditable withholding tax (CWT) on its purchases from domestic
suppliers.

50. The value-added tax payable on the domestic sales to private


entities is:
a. Zero
b. (P10,000)
c. P60,000
d. P70,000

51. The total input taxes attributable to zero-rated sales


is:
a. P20,000
b. P30,000
c. P50, 000
d. P60, 000

52. If the input taxes attributable to zero-rated are claimed as


tax credit, the net input value-added tax refundable is:
a. Zero
b. P40,000
c. P50,000
d. P60,000

53. The actual input taxes attributable to the domestic sales to


the government is:
a. P25,000
b. P30,000
c. P55,000
d. P65,000

54. The value-added tax payable on the domestic sales to the


government which was withheld as final withholding VAT is:
a. P5,000
b. P25,000
c. P30,000
d. P35,000

55. The journal entry to take up the domestic sales to the


government is:
a. Cash/AR 560,000
Sales 560,000
b. Cash/AR 560,000
Sales 500,000
Output tax 60,000
c. Cash/AR 535,000
Final withholding VAT 25,000
Sales 500,000
Output tax 60,000
d. Cash/AR 530,000
Final withholding VAT 25,000
Creditable withholding tax 5,000
Sales 500,000
Output tax 60,000

Private Entities
VAT 0% VAT Government
OT 500,000 x 12% 60,000 60,000
500,000 x 0% -0-
IT (30,000) (20,000)
500,000 x 7% (35,000)

120,000 x 1 / 4 (30,000) (30,000)


VAT Payable -0-
Excess Input Tax (50,000)
FINAL VAT Withheld 25,000

56. The journal entry to reflect the excess of actual input VAT-
over the statutorily allowed input tax on the domestic sales to
the government is:
a. Output tax 60,000
Revenue and expense summary 20,000
Input tax 55,000
Final withholding VAT 25,000
b. Output tax 60,000
Input tax 15,000
Final withholding VAT 25,000
Revenue and expense summary 20,000
c. Output tax 60,000
Input tax 55,000
Revenue and expense summary 5,000
d. Output tax 60,000
Revenue and expense summary 20,000
Input tax 55,000
Cash/AP 25,000

57. Which one among the following statements is wrong?


a. Private Banks may be authorized to collect internal revenue taxes.
b. The Bureau of Customs (BOC) is also charged with the collection
of internal revenue taxes.
c. The local government unit’s (LGUs), such as cities,
Municipalities
and Provinces, from part of the national tax system.
d. The bureau of Internal Revenue (BIR) is part of the administrative
machinery for the assessment and collection of internal
revenue taxes.

58. The prescriptive period for the issuance of a formal letter


of demand and final assessment notice (FLD/FAN) may not
ordinarily be stayed because of the lifeblood theory.
There are certain instances, however, where the running of
the prescriptive period
may be suspended. Which among the following instances is not
among the recognized exceptions which suspend the prescriptive
period within which to assess?
a. If the taxpayer is out of the country.
b. If the taxpayer changes his address, informing the commissioner
of
such change.
c. Where the taxpayer request for and is granted a re investigation
by the Commissioner.
d. When the Commissioner of Internal Revenue (CIR) is prevented
from making an assessment and within 60 days thereafter.

59. An examination of a calendar year corporate taxpayer’s


records shows that it filed its final adjustment income tax
return on February 15, 2016 for its 2015 income. It
subsequently filed an amended income tax return March 21, 2016.
Up to what date is the Bureau of Internal Revenue (BIR)
within which to issue a formal letter of demand and final
assessment notice (FLD/FAN)?
a. December 31, 2018 c. March 21,
2019
b. February 15, 2019 d. April 15,
2019

60. A closely held corporation has initially offered its shares in


the Philippine stock exchange (PSE). The following data pertain
to the initial public offering (IPO):

Number of shares sold in IPO 1,000,000


shares
Total outstanding shares
before
the listing in the PSE 3,000,000
shares
Gross value in money of the IPO
P20,000,000
The percentage tax due
is:
a. P100,000 c. P400,000
b. P200,000 d. P800,000

Gross Value of IPO


20,000,000
OPT Rate 1M / 4M = 25% ( 4%, from 4%,2%, 1%) 4%
Percentage Tax 800,000

61. Shares of stock in a domestic corporation held as investment


when sold not through the local stock exchange shall be subject
to:
a. 3% OPT or 12% VAT based on gross income
b. ½ of 1% based on gross selling price or gross value in money.
c. 5% on first P100, 000 capital gain; 10% on excess of P100, 000.
d. 4%;2%;1% based on gross selling price or gross value in money.
62. Malakas and Maganda were legally separated. They have six
minor children, all qualified to be claimed as additional
exemptions for income tax purposes. The court awarded custody
of two of the children to Malakas and three to Maganda,
with Malakas directed provide full financial support for them as
well. The court awarded the sixth child to Malakas’ parents
with Malakas also providing full financial support.
Assuming that only Malakas is gainfully earning while
Maganda is not, for how many children could
Malakas claim as additional exemptions when he files his annual
income tax return?
a. Two children c. Five
Children
b. Three children d. Six
children

63. Statement 1. Health and/or hospitalization insurance premium


is deductible from gross income by the spouse who
claimed the additional exemptions in case of married income
taxpayers.
Statement 2. Health and/or hospitalization insurance premium
paid by an individual income taxpayer is deductible from gross
income for a minimum amount of P2, 400 provided the family’s
gross income for the year does not exceed P250, 000.
a. Both statements are true
b. Both statements are
false
c. Statement 1 is true while statement 2 is
false.
d. Statement 2 is true while statement 1 is
false.

64. The following fringe benefits were given by an employer to


its employees for the quarter ending September 30, 2016:

De minimis benefits (not exceeding the maximum) P 200,000


Reimbursed expense of rank and file employees 400,000
Housing benefits to managers and supervisors
(Representing total rents) 680,000

The fringe benefit tax payable for the quarter


is:
a. P160, 000 c. P442,
353
b. P320, 000 d. P502,
353

Housing Benefits to Manager


680,000 x 50% = 340,000 / 68% x 32% = 160,000

65. Continuing number 64, how much would be the total deductions
from gross income which may be claimed by the employer?
a. P1,100,000 c.
P1,440,000
b. P1,280,000 d.
P1,600,000

De minimis benefits (not exceeding the maximum) P 200,000


Reimbursed expense of rank and file employees 400,000
Housing benefits to managers and supervisors
(Representing total rents) 680,000
Fringe Benefit Tax Expense 160,000
Total Deductions from Gross Income 1,440,000
66. Going back to the number 64, the employer shall file a
remittance return of the final tax on fringe benefit and pay the
tax withheld within:
a. 5 days from the close of each
month. b. 10 days from the close of
each month.
c. 10 days from the close of each fiscal quarter.
d. 10 days from the close of each calendar quarter.
67. Kalansay, a native of Negros, died leaving a property acquired
by purchase from Naty Gok who died 3 ½ years ago. The property
is now a Kalansay’s gross estate. The estate’s vanishing
deduction rate is:
a. 0%
b. 20%
c. 40%
d. 60%
Property is not gratuitously acquired by the present
Estate

68. A lessor of residential units has the following gross receipts


for
2016:
Monthly Number
Rent per unit of units Total
P 9,000 5 P 540,000
10,000 5 600,000
11,000 5 660,000
12,000 5 720,000

The output VAT


is:
a. P0 c.
P270,000
b. P230,340 d.
P302,400
Leasing on a residential units is VAT
Exempt

69. Using the same facts in number 68, only that the lessor is a
VAT- registered person, the OPT tax due is:
a. P0 c.
P67,500
b. P57,585 d.
P75,600
OPT Exempt
also

70. The deduction allowed for the payment of premium on


hospitalization insurance during the taxable year by a resident
citizen amounting to P3, 000 for the months of August to December
is:
a. P 3,000
b. P2, 400
c. P1, 200
d. P1, 000
200 x 5 months =
1,000

71. The following are among the constitutional limitation on the


power of taxation. Which one is not?
a. Due process clause
b. Non-impairment clause
c. Equal protection clause
d. No imprisonment for non-payment of tax
72. Pacmom filed her income tax from return for 2015 on May 16,
2016 and paid the tax of P50,000. Upon audit by the BIR an
assessment notice was issued on April 30, 2019, requiring Pacmom to
pay a deficiency tax of P 75,000 not later than July 30, 2019.
Pacmom will:
a. False prescription as defense
b. Request for an extension of time to pay the deficiency income tax.
c. Go to the court of tax appeals to appeal the assessment made by
the
BIR.
d. Ignore the assessment as the date of collection is already way
beyond three years, covering taxable year 2015.

73. Date March 08, 2016


Date assessment was received
petition for
reinvestigation
was filed by the BIR March 18, 2016
Date of filing of documents to
support the petition April 08, 2016
Date decision of denial of the
petition was received April 28, 2016
The last day to appeal to the is on:
CTA

74. Using the same facts in number 73, only that instead of going
to the CTA, a request for reconsideration was filed with the
BIR on May 7, 2016. Date decision of denial of the request
for reconsideration was received on June 2, 2016. The last
day to appeal to the CTA is on:
a. June 7, 2016 c. June 23,
2016
b. June 21, 2016 d. July 2,
2016

75. Continuing number 74 only that instead of receiving a


decision denying a request for reconsideration, a revised
assessment was received on June 2, 2016. The last day to appeal
to the CTA is on:
a. June 7, 2016 c. June 23,
2016 b. June 21, 2016 d. July 2, 2016
30 days from June 2,
2016

76. Statement 1. The Bureau of Internal Revenue (BIR) has as its


powers and duties the assessment and collection of the national
internal revenue taxes, fees, and changes and the
enforcement of all fortitudes, penalties, and fines connected
therewith.

Statement 2. The Bureau of Customs (BOC) has its powers and


duties the assessment and collection of awful revenues from
imported and exported articles and all other dues, fees, charges,
fines, and penalties accruing under the tariff and customs laws.
a. Both statements are correct.
b. Both statements are wrong.
c. Statement 1 is correct but statement 2 is
wrong. d. Statement 2 is correct but statement 1
is wrong.
77. A customs duty that imposes both advance and specific customs
duties on imported or exported articles is:
a. Anti-dumping duty.
b. Countervailing
duty. c. Discriminating
duty.
d. Compound customs duty.

78. Consider the following statements:


i. The imposition of customs duties also assists in
economic development.
ii. Customs duties are sometimes imposed to protect local
consumers.
iii. The purpose of regular customs duties is to raise
revenues to meet the needs of government.
iv. Compound customs duties are computed only on the basis
of units of measure, such as weight, measurement,
quantity, etc.
a. All the above statement is correct.
b. Only statements I, II and III are correct.
c. Only statements I, II and IV are
correct. d. Only statement I, III and IV
are correct.
Numbers 79 to 80 are based on the following information pertaining to
taxable year 2016: Phil. USA
Sales P500,000 P600,000
Sales discounts,
Returns and allowances 20,000 50,000
Cost of sales 230,000 250,000
Rent expenses 50,000 80,000
Salaries and wages 50,000 60,000
Interest expenses 10,000 -
Entertainment and presentation 10,000 -
Contribution to charitable institution 10,000 -
Interest on bank deposit 10,000 20,000
Royalty- musical compositions 20,000 20,000
Dividends from domestic corporation 10,000 -

79. The taxable income if the taxpayer is a domestic corporation


(DC)
and resident citizen (RC), under:

Itemized Deductions Optional Deduction


DC RC DC RC
a. P320,000 P270,000 P330,000
P280,000 b. P328,150 P278,150
P354,000 P592,000 c. P280,000 P230,000
P378,000 P328,000 d. P288,150 P238,150
P330,000 P568,000

ITEMIZED DEDUCTIONS DC
RC
Passive Income 40,000
40,000

Sales 1,100,000 1,100,000


Sales Disc, Ret & Allowances (70,000) (70,000)
Net Sales 1,030,000 1,030,000
Cost of Good Sold (480,000) (480,000)
Business Gross Income 550,000 550,000

Total Gross Income 590,000


590,000
Other Expenses (240,000)
(240,000) Interest 10,000 - (10,000 x 33%) (6,700)
(6,700) Entert & Repre (1,030,000 x .5%) (5,150)
(5,150) Net Income Before CC 338,150
338,150

CC Actual < Limit (10,000)


(10,000) Personal Exemption -0-
(50,000) Taxable Income 328,000
278,150

OPTIONAL DEDUCTION DC RC
Passive Income 40,000 40,000
Net Sales 1,030,000
Business Gross Income 550,000
Total Gross Income/
Gross Receipt & Net Sales 590,000 1,070,000
40% OSD (236,000) (428,000)
Personal Exemption -0- (50,000)
Taxable Income 354,000 592,000

80. The taxable income if the taxpayer is a resident foreign


corporation (RFC) and non-resident citizen (NRC):
RFC NRC
a. P160,000
P270,000
b. P130,900 P
80,900 c. P133,855
P 80,900 d. P120,000
P 70,000

ITEMIZED DEDUCTIONS RFC


NRC
Passive Income
Sales 500,000
500,000

Sales Disc, Ret & Allowances (20,000) (20,000) _


Net Sales 480,000 480,000
Cost of Good Sold (230,000)
(230,000) Other Expenses (100,000)
(100,000) Interest 10,000 - (10,000 x 33%) (6,700)
(6,700) Entert & Repre (1,030,000 x .5%) (2,400)
(2,400) Net Income Before CC 140,900
140,900
CC 140,900
Personal x 5%
Exemption (7,045)
-0- (10,000)
(50,000)
Taxable Income 133,855 80,900
TAXATION QUIZZER PART 3

1. The following are common to the inherent power of taxation,


power of eminent domain and police power, except for which of the
following?
a. They are necessary attributes to the
sovereignty. b. They interfere with private rights
and property. c. They affect all persons or the
public.
d. They are legislative in implementation.

2. In case of ambiguity, tax laws shall be interpreted:


a. Strictly against the taxpayer.
b. Liberally against the government.
c. Liberally in favor of the taxpayer.
d. Liberally in favor of the government.

3. Which of the following statements is not correct?


a. Taxes may be imposed to raise revenue or to provide
incentives or disincentives for certain activities within the
state.
b. The state can have the power of taxation even if the
constitution does not expressly give it the power to tax.
c. For the exercise of the power of taxation, the state can
tax anything at anytime.
d. The provisions of taxation in the Philippine Constitution
are
grants of power and not limitation on the taxing power.

4. Which of the following is not a direct tax?


a. Immigration tax c. Income tax
b. Transfer tax d. Contractor’s tax (Now VAT)

5. In which situation will a CPA’s signature be necessary?


a. When the value of the gross estate is P2,000,000 and
above. b. When the value of the gross estate exceeds
P2,000,000.
c. Regardless of the value where the gross estate consists or
registered or registrable property.
d. When the value of the gross estate exceeds P200,000,
although
exempt from transfer tax.

6. A seller of goods is non-VAT registered. His annual gross sales


amount to P1,919,500. To what business tax is he liable?
a. 3% tax on VAT-exempt persons c. 3% common carrier’s tax
b. 12% value-added tax d. Not subject to any percentage
tax

7. Abigail sold through the local stock exchange, 10,000 PLDT shares
that she bought 2 years ago. Abigail sold the shares for P2
million and realized a net gain of P200,000. The transactions is
a. Subject to regular income tax rates for individuals but only
50%
shall be recognized because it is a long-term capital
gain. b. Subject to capital gains tax amounting to P15,000.
c. Subject to percentage tax amounting to P10,000.
d. Subject to regular income tax rates for individual under
Section
24 (A).
8. Proceeds of life insurance to the extent of the amount receivable
by the estate of the deceased, his executor or administrator
under policies taken out by the dependent upon his own life shall be
I. Part of the gross estate irrespective of whether
or
not the insured retained the power of
revocation.
II. Not part of the gross estate if the beneficiary is
irrevocable.
III. Part of the gross income if the designation of the
beneficiary is revocable.
IV. Not part of the gross income irrespective of
whether
or not the insured retained the power of
revocation. a. I and II
b. I and
III c. I and
IV d. only I

9. One of the following is not correct


Deduction Maximum
a. Funeral Expenses P200,000
b. Family home P2,000,000
c. Medical expenses P500,000
d. Standard Deduction P1,000,000

10. One of them is not considered non-resident citizen?


a. A citizen of the Philippines who establishes to the
satisfaction of the Commissioner the fact of his physical
presence abroad with a definite intention to reside therein.
b. A citizen of the Philippines who leaves the Philippines
during the taxable year to reside abroad, either as an immigrant or
for employment on permanent basis.
c. A citizen of the Philippines who works and derives income
from abroad and whose employment thereat requires him to be
physically present abroad most of the time during the
taxable year.
d. A citizen of the Philippines who went on a business trip
abroad
and stayed therein most of the time during the year.

11. Z is a Filipino immigrant living in the United States for more than
10 years. He is retired and he came back to the Philippines
as a balikbayan. Every time he comes to the Philippines, he stays
here for about a month. He regularly receives a pension from
his former employer in the United States, amounting to USD1,000 a
month. While in the Philippines, with his pension pay from his
former employer, he purchased 3 condominium units in Makati, which
he is renting out for P15,000 a month each. Does the USD1,000
pension become taxable because he is now in the Philippines?
a. Yes. Income received in the Philippines by non-resident
citizens is taxable.
b. Yes. Income received in the Philippines or abroad by
non- resident citizens is taxable.
c. No. Income earned abroad by non-resident citizens is no
longer taxable in the Philippines.
d. No, the pension is exempt from taxation being one of the
exclusions from gross income.

12. Which of the following income earners is required to file income


tax return?
a. Minimum wage earners
b. Non-resident alien not engaged in business
c. An individual with respect to pure compensation income
deriving from such sources within the Philippines, the
income tax on which has been correctly withheld and
that an individual deriving compensation from one employer
at any time during the taxable year
d. General professional partnership

13. Which of the following fringe benefit is not subject to fringe


benefit tax?
a. a. Housing benefit c. De minimis
benefit b. b. Expense account d.
Vehicle benefit

14. In the case of a taxpayer, only the following percentages of the


gain or loss recognized upon the sale or exchange of a capital
asset shall be taken into account in computing net capital gain,
net capital loss and net income.
I. Statement 1 One hundred percent (100%) if the capital
asset has been held for not more than twelve (12)
months by a taxpayer.
II. Statement 2 Fifty percent (50%) if the capital asset has
been held for more than twelve (12) months by a taxpayer.
a. True, true c. False, false
b. True, false d. False, true

15. 15.A, worked for a manufacturing firm but due to business


reverses, the firm offered a voluntary redundancy program in order
to reduce overhead expenses. Under the program, an employee
who offered to resign would be given separation pay equivalent to
his 3 months basic salary for every year of service. A accepted
the offer and received P800,000 as separation pay under the program.
After all employees who accepted the offer were paid, the firm
found its overhead still excessive. Hence, it adopted
another program, where various unprofitable departments were
closed. As a result, B was separated from the service. B also
received P800,000 as separation pay. At the time of separation
both A and B have rendered at least 10 years of service but A
was 55 years old while B was only 45 years old. As a result,
a. Both amounts are exempt from income
tax b. Both amounts are subject to income
tax c. Only Mr. A is subject to income
tax
d. Only Mr. B is subject to income tax
16. The following shall not be subject to estate tax
a. The merger of usufruct in the owner of naked title
b. The transmission or delivery of inheritance or legacy by the
fiduciary heir or legatee to fideicommissary
c. The transmission from the first heir, legatee, or donee in
favor
of another beneficiary in accordance with the desire of the
predecessor
d. All bequest, devises, legacies or transfers in social
welfare, cultural and charitable institutions

17. Which of the following transactions is subject to zero-rated


value- added tax?
a. Services rendered to persons engaged in international
shipping or air transport operations
b. Services rendered banks, non-bank financial
intermediaries c. Generation, transmission and distribution
of electricity
d. Services rendered by professionals such as CPA, Physicians,
and
Lawyers.

18. Which of the following businesses is allowed presumptive input


value- added tax?
a. Manufacturer of canned goods
b. Manufacturer of packed juices
c. Manufacturer of packed
noodles d. Manufacturer of dried
fish

19. Which of the following shall be allowed as a deduction from the


gross income?
a. Depreciation for vehicle for land transport used for personal
purposes the value of which does not exceed P2,400,000.
b. Depreciation for vehicle for land transport used in the
business the value of which exceeds P2,400,000.
c. Depreciation for land vehicles the value of which
exceeds P2,400,000 where the taxpayer’s main line of
business is transport operations or lease of transportation
equipment and the vehicles purchased are used in said
operations.
d. Depreciation for airplanes and/or aircraft the value of which
exceeds P2,400,000 where the taxpayer’s main line of business
is sale of goods or properties and the vehicles purchased are
used in said operations.

20. A, at the time of retirement, had 1,000 pieces of merchandise


which was deemed sold at a value of P20,000, with an output tax of
P2,400. After retirement, A sold to B 500 pieces of these for
P12,000. In the contract of sale or invoice, A stated sales invoice
number wherein the output tax on deemed sale was imposed and the
corresponding tax paid on the 500 pieces. He prepared the following
invoice:
I. Gross selling price P10,800
II. VAT previously paid on deemed sale 1,200
III. Total P12,000
How much is the input tax of
B?
a. P2,400
b. P1,200
c. P1,000
d. None of the
above

21. If a VAT-registered person issues a VAT invoice or VAT


official receipt for VAT-exempt transaction, but fails to display
prominently on the invoice or receipt the words “VAT-exempt sale”, the
transaction shall:
a. Still be exempt from value-added tax
b. Become taxable and the issuer shall be liable to pay VAT
thereon c. Be effectively subject to zero percent
d. Be considered erroneous transaction and must be disregarded.

22. Which of the following franchise grantees is subject to the 2%


percentage tax on franchise?
a. Franchise on radio and/or television broadcasting companies
the gross annual receipts in the preceding year do not exceed
P10,000,000.
b. Franchise on gas and water
utilities. c. Franchise on toll road
operations.
d. PAGCOR and its licenses and franchisees.

23. Which of the following is subject to value-added tax?


a. Medical, dental, hospital, and veterinary
services. b. Laboratory services.
c. Sale of drugs and medicines in the hospital’s pharmacy or
drug store.
d. Medicine administered to a patient confined in the hospital.

24. Which of the following statements is incorrect regarding


standard deduction?
a. A deduction in the amount of P1,000,000 shall be allowed as
an additional deduction without need of substantiation.
b. The full amount of P1,000,000 shall be allowed as deduction
for
the benefit of the
decedent.
c. Standard deduction is a deduction from the conjugal or community
properties of married decedents.
d. Standard deduction is not allowed to decedents who are non-
residents alien.

25. The actual residential home of the decedent and his family at the
time of his death, as certified by the Barangay Captain of the
locality where the family home is situated has a fair
market value of P1,500,000. The family home is part of the common
property of the spouses. How much is the family home deduction?
a. a. P1,500,000 c. P750,000
b. b. P1,000,000 d. P500,000
26. Who makes revenue regulations?
a. Secretary of of
Finance c.
b. Commissioner BIR d. Board
House of
of Accountancy
Reperesentatives
27. Diety is non-stock, non-profit organization made an importation
of agricultural product in its original state from a Chinese
farmer amounting to P2,240,000. If you are the customs collector how
will you treat such importation, will it be?
a. a. Subject to 12% VAT c. Exemption from VAT
b. Subject to zero rated VAT d. Subject to 3% percentage tax

28. George and Pearl were legally separated. They had six minor
children, all qualified to be claimed as additional exemptions for
income tax purposes. The court awarded custody of two of the
children to George and three to Pearl, with George directed to
provide full financial support for them as well. The court awarded
the 6th child to George’s father with George also providing full
financial support. Assuming that only George is gainfully employed
while Pearl is not, how much could George claim as additional
exemptions when he files his income tax return?
a. a. P100,000 c. P50,000
b. b. P75,000 d. None

29. Construction by XYZ Construction Co. of concrete barrier for the


Asian Development Bank in Ortigas Center to prevent car bombs from
ramming the ADB gates along ADB Avenue in Mandaluyong City is
subject to what business tax?
a. a. 12% VAT c. Exemption from VAT
b. b. 0% VAT d. None of the above
30. a number of inquiries from shareholders, James Hong
has
issued a formal announcement that his company is doing well.
a. Following b. Beside c. Against d. Toward

31. Mr. Araki, a non-resident alien stockholder, receives a


dividend income of P300,000 in 2012 from a foreign corporation
doing business in the Philippines. The gross income of the foreign
corporation from within and without the Philippines for three years
preceding 2012 are
as follows: 2009 2010 2012
From w/in the Philippines P16,000,000 P12,000,000 P14,000,000
From without the Philippines 18,000,000 14,000,000 16,000,000

How much of the dividend income received by Mr. Araki is


considered income from sources within the Philippines?
a. Zero b. P150,000 c. P270,000 d. P300,000
32. Martin Brian is an independent travelling salesman, deriving
his income solely from commissions and personally bearing all expenses
without reimbursement or any kind. During the taxable year
2016, Martin Brian incurs the following expenses pertaining directly to his
activities as an independent salesman:

Travelling expenses P650,000


Secretarial expenses 250,000
Long-distance telephone bills 150,000
Freight expenses for products sold 300,000
Insurance for product transported 50,000
Life insurance premiums 25,000 do not include
Doctor’s fee incurred while he was on 15,000 do not
include one of his sales trip

What amount can Martin Brian deduct from his gross


income?
a.
P1,440,000
b. P1,400,000
c. P1,150,000
d. P40,000

33. A mode of acquisition by virtue of which, the property, rights


and obligations, to the extent of the value of the inheritance of a
person are transmitted through his death to another either by his will or by
operation of law
a. Extinctive prescription c. Succession
b. Acquisititive prescription d. Donation mortis causa

34. In 2016, Claudio, a self employed resident citizen, paid


the following:
Philippine income tax for 2016 P1,000,000
Professional tax 300
Gravel and sand tax 20,000
U.S.A income tax 2016 500,000
Real property tax on his house 3,000
Road user’s tax on his delivery vans 50,000
Local annual fixed tax for his delivery vans 10,000
Interest for late payment of national & local taxes 40,000
Surcharge for late payment of national & local taxes 60,000

If Claudio had signified in his return to avail of tax credit of


foreign income tax paid, how much deduction may he claim on his 2016
income?
a. P1,583,300 c.
b. P583,300 d. P580,300
P80,300

35. Rose Company provided fringe benefit to its managerial employees


in the amount of P136,000 and to its rank and file employees amounting to
P50,000. The deductible expense by Rose Company is:
a. P250,000 c. P136,000
b. P186,000 d. P50,000
36. The producers of the new play After Dusk decided to add more
performances because of the positive response to early
shows.
a. Overwhelm c. Overwhelmed
b. Overwhelming d. Overwhelmingly

37. Estate tax is


a. An tax c. A percentage tax
b. An excise
income tax d. A sales tax

38. The head priest of the religious sect Tres Personas Solo Dios as
the corporation sole, rented out 5,000 sq.m. lot registered in
its name for use as school site of a school organized for profit. The
sect used the rentals for the support and upkeep of its priests.
The rented lot is
a. Not exempt from real property taxes because the user is
organized for profit.
b. Exempt from real property taxes since it is actually,
directly and exclusively used for religious purposes.
c. Not exempt from real property taxes since it is the rents
not the land that it used for religious purposes.
d. Exempt from real property taxes since it is actually,
directly and exclusively used for educational purposes.

39. Which of the following compensation of minimum wage earner shall


be taxable?
a. Holiday pay c. Night shift differential
pay b. Overtime pay d. Commission pay

40. Which principle of sound tax system suggests that the sources
of revenue should be sufficient to meet the requirements
of the government expenditures?
a. Fiscal adequacy c. Theoretical justice
b. Administrative feasibility d. Uniformity of
taxes

41. The following are the powers of the Bureau of Internal


Revenue, except:
a. The assessment and collection of all national internal
revenue taxes.
b. The execution of all judgments in all cases decided in its
favor by CTA and the ordinary courts.
c. The enforcement of all forfeitures, penalties, and fines
connected with national revenue taxes.
d. The assessment and collection of customs duties and tariffs.

42. Statement 1 An individual taxpayer is allowed to claim an


optional standard deduction of 40% of gross income.
Statement 2 A corporate taxpayer is allowed to claim optional
standard deduction of 40% of gross income.
a. a. True, true b. True, false c. False, true d. False,
false
43. Vivian is a VAT registered grocery owner and sugar dealer.
She submitted lists of inventory as of December 31, 2016 to the
Revenue District Officer as follows:
Grocery items, total value P325,000
Raw cane sugar, total value 255,000

In January 2017, she had the following sales and purchases:


Sales Purchases
Grocery (total invoice value) P1,042,525 P 650,100
Raw cane sugar (excluding VAT) 480,000 420,000

The VAT due for January 2017 is


a. P6,675.26 b. P35.675.00 c. P41,675.31 d. P42,045.54

**(1,042,525 X 12%/1.12) - (650,100 X 12%/1.12) = 42,045,54

44. Which is correct?


a. Basic personal exemption of a single individual is
P20,000. b. Basic personal exemption of a head of the family
is P25,000. c. Basic personal exemption of married individual
is P32,000.
d. Additional personal exemption per qualified dependent is
P25,000.

45. This shall be considered a livelihood activity only and shall not
be considered doing business, if the gross sales or receipts in one
year does not exceed
a. P100,000 c. P200,000
b. P550,000 d. P60,000

46. Antoine Bee is the operator of Globe Bee Coliseum. During the
month, it had the following gross receipts from various activities,
to wit:
I. Concert by April Boy Ireneo P85,000
II. Professional Basketball game 120,000
III. Amateur basketball games 36,700
IV. Billiards 45,000
The percentage tax payable by Antoine Bee on the Coliseum is:
a. P18,000 c. P24,750
b. P23,505 d. P43,005
120,000 x 15% =
18,000

47. As franchise. Ms. L.A. Young had the following data on revenues
and receivables, taxes not included:
Revenue A/R, beginning A/R, ending
From operations:
Covered by franchise P2,000,000 P300,000 P400,000
Not covered by the franchise 600,000 50,000

How much is the percentage tax, if operating a water system?


a. P38,000 b. P52,000 c. P90,000 d. P190,000
300,000 + 2,000,000 - 400,000 = 1,900,000 x 2% = 38,000
Items 48 – 52 are based on the following information:
The decedent, a married man with surviving spouse under conjugal
partnership of gains, with the following data died on March 1, 2016.

Real Properties:
Family lot acquired by the decedent before marriage, FMV P 500,000

Family house built using the salary earned by the surviving


spouse during the marriage, FMV 900,000

Coconut farm inherited by the surviving spouse during the


marriage, FMV 100,000

Mango orchard, FMV 800,000

Personal Properties:
Bank deposit under the name of the decedent
representing salary earned before marriage P2,150,000

Gold necklace inherited by the surviving spouse


during the marriage 120,000

Several pieces of jewelry acquired during the marriage using


the exclusive money of the decedent 300,000

The following expenses/deductions are claimed:


Funeral expenses (40% shouldered by relatives) 100,000
Judicial expenses (including P50,000 lawyer’s fee incurred
on September 5, 2016) 120,000
A piece of jewelry lost through theft on August 15, 2016 50,000
Gambling debts of the decedent 70,000
Unpaid realty tax (coconut farm) for the 3 quarters of 2016) 150,000
Claims against an insolvent debtor (25% can be collected) 80,000
Medical expenses 600,000

48. How much is the conjugal properties?


a. P1,780,000
b. P1,700,000
c. P900,000
d. None of the
choices

49. How much is the exclusive


properties?
a. P2,950,000 c. P450,000
b. P650,000 d. None of the choices

50. How much is the family home deduction?


a. P1,000,000 c. P700,000
b. P950,000 d. None of the choices

51. How much is the total ordinary deductions?


a. P460,000 c. P260,000
b. P310,000 d. None of the choices
Funeral Expense(100,000 x 60%) 60,000
Judicial Expenses 120,000
Jewelry theft 50,000
Claims against Insolvent person 80,000
Total Ordinary Deductions 310,000

52. How much is the taxable net estate?


a. a. P1,225,000 c. P1,125,000
b. b. P1,175,000 d. None of the choices

Items 53 - 56 are based on the following information:

Jon Asty, single resident citizen (who got married during the year
2016), has the following dependents for the taxable year 2016.

Leo, adopted child, 25 years old, person with disability, dies during
the year

Dax Niel, child of his wife in a previous marriage, 18 years old

John Denver, acknowledged natural child, becomes 21 years on January 1,


2016

John Rey, child by natural adoption, 15 years old

Efren, born December 31, 2016 (taxpayer and his wife’s first born child)

The following information pertain to his income and expenses in the


year
2016:
Salary, gross of P8,500, withholding tax P220,000
Rent expense, apartment house 36,000
Health insurance premium paid (P500/month starting August) 2,500
Gross sales from trading business 150,000
Cost of sales 120,000
Itemized business expenses 10,000

53. The
a. total personal exemptions shall be:
a. P150,000 c. P75,000
b. b. P100,000 d. P60,000

54. The deductible health insurance premium paid is:


a. a. P2,400 c.
b. b. P1,200 d. P1,000
None

55. The taxable net income using itemized deduction is:


a. a. P164,000 c.
b. b. P160,000 d. P189,000
P89,000

56. The taxable net income using optional standard deduction is:
a. P164,000 c.
b. P160,000 d. P189,000
P89,000
Items 57 - 61 are based on the following information:
Joefrey Ochoa Manufacturing Corp. is a VAT-registered enterprise which
is also engaged in VAT-exempt transactions. It has the following data
taken from its books of accounts for the first quarter of the fiscal year
(May 1,
2016 – April 30,
2017)

Domestic sale of goods


P5,000,000
VAT-Exempt sale of goods
1,000,000
Domestic purchases of goods for use in VAT-subject transactions
500,000
Importation of goods 800,000
Domestic purchases of for use infor
services VAT-subject transactions
all transactions 450,000
Purchase of vehicles for land transport for use in all
transactions
(estimated life 5 years, purchased May 1, 2016) 2,400,000
Maintenance expense for the vehicle for land transport 100,000
Other operating expenses 500,000
Monthly VAT paid (May & June) 300,000

57. How much is the creditable input tax?


a. a. P460,000 c. P201,000
b. b. P223,000 d. None of the choices

58. How much is the VAT payable?


a. a. P140,000 c. P387,000
b. b. P377,000 d. P99,000

59. How much is the taxable net income?


a. P3,741,000 c. P3,170,000
b. P3,530,000 d. P2,530,000
60. When shall the VAT return be filed for the first fiscal quarter?
a. August 15, 2016 c.
b. August 20, 2016 d. August 25,29,
September 2016
2016
61. When shall the income tax return be filed for the first fiscal
quarter?
a. August 25, 2016 c. September 30, 2016
b. September 29, 2016 d. None of the
choices

Items 62 - 64 are based on the following information:


Jason Elcano Corporation has a soft spot for senior citizens and
persons with disability (PWDs). As such, it hires senior citizens and
PWDs to work in the company for at least six (6) months. The following
data taken from the books of accounts that are provided by the
Corporation:
Gross sales
P15,000,000
Cost of sales
5,000,000
Salaries of senior citizens
500,000
Salaries of PWDs
300,000
Actual amount of assistance under Adopt-A-
School
program (fully deductible)
200,000
Other operating expenses
3,000,000
During the same period the corporation also allows 20% discount to
senior citizens and PWDs who buy goods from the company. The sales are
as follows (not part of the gross sales above).
Sales to senior citizens P
800,000
Sales to PWDs
500,000
Total
P1,300,000

62. How much is the total discount given?


a. a. P260,000 c. P100,000
b. b. P160,000 d. Answer not given

63. How much is the total itemized deductions including the


special itemized deductions?
a. a. P4,480,000 c. P4,175,000
b. b. P4,435,000 d. Answer not given

64. How much is the output VAT of the corporation?


a. a. P2,079,456 c. P1,828,000
b. b. P1,848,000 d. None of the choices

65. The proceeds received under a life insurance endowment contract is


not considered part of the gross income
a. If it is so stated in the life insurance endowment
policy b. If the price for the endowment policy was not
fully paid
c. Where payment is made as a result of the death of the insured
d. Where the beneficiary was not the one who took out the
endowment contract
66. When shall the fringe benefits tax return be filed?
a. On or before 10th day of the month following the quarter in
which
the withholding was made.
b. On or before 10th day of the month following the withholding
was
made.
c. On or before 15th day of the month following the quarter in
which
the withholding was made.
d. On or before 15th day of the month following the withholding
was
made.

67. A, an individual, sold to B, his brother-in-law, his residential


lot with market value of P1,000,000 for P600,000. A’s cost in the
lot is P100,000. B is financially capable of buying the lot. What
tax should be imposed and collected from A as a result of the
transaction?
a. Presumed capital gains tax c. Real property tax
b. Donor’s tax d. Tax on the transfer of
real property
68. Which of the following may not raise money for the government?
a. Power of taxation c. Power of eminent domain
b. Police power d. Privatization of government’s
capital assets
69. Riyadh Motors has succeeded generating positive publicity for
its new
line
of
ecol
ogic
ally
frie
ndly
auto
mobi
les.
a.

On
b
.

I
n

c
.

A
t

d
.

O
f

70. The auditor d.


has Many
a. Much eviden
evidence ces
b.
b.
Ma
ny
ev
id
en
ce
c.
c.
Mu
ch
ev
id
en
ce
s
d.
to warrant a
full blown
forensic audit.

You might also like